Criminal Law Practice Questions

अब Quizwiz के साथ अपने होमवर्क और परीक्षाओं को एस करें!

A threatens B with death unless B robs a store. B robs the store. B is charged with robbery. "Robbery" is defined as "taking another person's property by force." Has B acted voluntarily within the meaning of the voluntary act requirement? A. Yes. B. No.

A

C is standing on a target range. C aims at the target and pulls the trigger. At that moment, D steps in front of the target. C causes D's death. C is charged with murder. "Murder" is defined as "intentionally causing the death of another human being." Has C acted voluntarily within the meaning of the voluntary act requirement? A. Yes. B. No.

A

V is drowning in the ocean. D, nearby, swims to V's aid. As a result, other nearby good samaritans, who were on their way to help, go about their own business. At the last moment, D decides not to help V, who then drowns. Does actus reus exist? A. Yes. B. No.

A

The accused is charged with the burglary of his neighbor's home. Evidence presented at the accused's trial indicates that he talked his girlfriend into assisting him. The jury is instructed on the common-law provisions relating to burglary, solicitation, conspiracy, and attempt. If the accused is found by the jury to be guilty of burglary: (A) He also may be found guilty of conspiracy, but not of solicitation or attempt. (B) He also may be found guilty of conspiracy and solicitation, but not of attempt. (C) He also may be found guilty of conspiracy or solicitation but not both, and he may not be found guilty of attempt. (D) He also may be found guilty of solicitation or attempt but not both, and he may not be found guilty of conspiracy.

A The accused may also be found guilty only of conspiracy if he is found guilty of burglary. One who solicits another to commit a crime cannot be convicted of both the solicitation and the completed crime. Likewise, one who completes a crime after attempting it may not be convicted of both the attempt and the completed crime. However, if conspirators are successful, they can be convicted of both criminal conspiracy and the crime they committed pursuant to the conspiracy (i.e., conspiracy does not merge with the completed offense). Thus, if the accused is found guilty of burglary, he cannot also be convicted of either attempt or solicitation. The accused can be convicted of conspiracy in addition to burglary (with conspiracy liability being based on the apparent agreement between the accused and his girlfriend to bring about the burglary of the neighbor's home). (B) incorrectly states that the accused may be found guilty of solicitation as well as the principal offense of burglary. (C) and (D) make the same incorrect assertion. In addition, (D) incorrectly states that the accused may be found guilty of attempt as well as the completed crime, and that he may not be found guilty of conspiracy

A woman wearing a fur coat stopped at a remote gas station to fill her gas tank. A man approached the woman with a knife and told her that if she did not give him the coat, he would stab her. The woman was alarmed and accidentally spilled gasoline on her fur coat as she stopped pumping gas and turned to face him. She then took off her fur coat and gave it to the man. The woman drove away, and the man put on the fur coat, sat down next to gas station, and lit a cigarette. The fur coat immediately caught on fire, and although the man was able to take the coat off before he was severely injured, the flames quickly spread. The gas station went up in flames and the gas attendant died in the fire. Police quickly apprehended the man. For which crime(s) can the man be charged? A. Robbery B. Arson and robbery C. Robbery and felony murder D. Arson and felony murder

A . Here, the man has committed robbery. Robbery is larceny from the person or presence of the victim by force or intimidation. All of the elements of larceny are necessary for robbery. Larceny is the trespassory taking and carrying away of the personal property of another with the intent to steal. The man took the woman's fur coat from her by threatening to stab her with his knife, meeting all requirements for robbery. However, the man is not guilty of arson. Arson is the malicious burning of the dwelling of another. The man did not maliciously burn the dwelling of another; he accidentally set fire to a gas station. For this reason, answer choices B and D are incorrect. Answer choices C and D are also incorrect. Felony murder is an unintended killing proximately caused by and during the commission or attempted commission of an inherently dangerous felony. Arson and robbery are inherently dangerous felonies, and an underlying felony will generally "merge" into the crime of felony murder for the purposes of Double Jeopardy. However, the gas station did not catch on fire during the commission of the robbery, so that felony cannot serve as the basis for a felony murder charge for the gas attendant's death. Rather, the man accidentally set the gas station on fire when he lit the cigarette after the robbery. As discussed above, this was not a case of arson. Thus, he is not guilty of felony murder.

A guest at a party becomes enraged over the host's advances towards the guest's wife. Threatening to burn down the host's home, the guest sets a chair on fire. In addition to consuming the chair, the fire causes extensive smoke damage to the room before it is put out. The guest is charged with common-law arson. Should he be convicted? A. No, because the fire did not damage the home. B. No, because the guest did not enter the home with the intent to set it on fire. C. Yes, because the structure was the host's home. D. Yes, because the guest had ill will towards the host.

A . In order for a fire set in a dwelling to be arson, there must be some damage (e.g., charring) to the structure of the dwelling. Smoke damage alone is insufficient. Answer choice B is incorrect because, unlike burglary, arson does not require that defendant enter the structure with the intent to commit a crime. Answer choice C is incorrect because, even though the structure where the fire took place was the dwelling of another, there was no damage to its structure. Answer choice D is incorrect because the guest's ill will towards the host may explain his motive for acting, but is not required to establish malice. Here, the guest's threat coupled with his act would likely be sufficient to establish the necessary malice.

A woman purchased a wardrobe closet at an antique auction. Three days later, while cleaning the inside of the closet, she discovered a small quantity of a white powder inside a box. She showed the box to her boyfriend, a paralegal. He identified the powder as driscamine, a controlled substance. He told her that it was illegal to buy driscamine, but because she did not know that it was in the closet when she purchased it, it was okay to keep it, which she did. A state statute prohibits "willful and unlawful possession of a controlled substance." If the woman is charged with violating this statute, should she be found guilty? A. Yes, because she knowingly possessed the driscamine. B. Yes, because she acquired the driscamine when she intentionally purchased the wardrobe closet and, in doing so, committed the requisite unlawful act. C. No, because she thought she was acting lawfully. D. No, because she did not willfully acquire the driscamine and, hence, committed no unlawful act.

A . She should be found guilty. As soon as her boyfriend informed her that driscamine was a controlled substance and she decided to keep it, she violated the statute, because she willfully possessed a controlled substance. (B) is incorrect because the statute does not punish mere possession; the possession must be "willful." (C) is incorrect because ignorance of the law is generally no excuse. It clearly does not negate the mental state required for this statutory crime. (D) is incorrect because the statute does not punish willful acquisition but willful possession.

The accused is charged with the burglary of his neighbor's home. Evidence presented at the accused's trial indicates that he talked his girlfriend into assisting him. The jury is instructed on the common-law provisions relating to burglary, solicitation, conspiracy, and attempt. If the accused is found by the jury to be guilty of burglary: A. He also may be found guilty of conspiracy, but not of solicitation or attempt. B. He also may be found guilty of conspiracy and solicitation, but not of attempt. C. He also may be found guilty of conspiracy or solicitation but not both, and he may not be found guilty of attempt. D. He also may be found guilty of solicitation or attempt but not both, and he may not be found guilty of conspiracy.

A . The accused may also be found guilty only of conspiracy if he is found guilty of burglary. One who solicits another to commit a crime cannot be convicted of both the solicitation and the completed crime. Likewise, one who completes a crime after attempting it may not be convicted of both the attempt and the completed crime. However, if conspirators are successful, they can be convicted of both criminal conspiracy and the crime they committed pursuant to the conspiracy (i.e., conspiracy does not merge with the completed offense). Thus, if the accused is found guilty of burglary, he cannot also be convicted of either attempt or solicitation. The accused can be convicted of conspiracy in addition to burglary (with conspiracy liability being based on the apparent agreement between the accused and his girlfriend to bring about the burglary of the neighbor's home). (B) incorrectly states that the accused may be found guilty of solicitation as well as the principal offense of burglary. (C) and (D) make the same incorrect assertion. In addition, (D) incorrectly states that the accused may be found guilty of attempt as well as the completed crime, and that he may not be found guilty of conspiracy.

A broker discovered that a coworker kept his very expensive collection of watches in a locked desk drawer in his office. Over lunch at a restaurant, the broker revealed this discovery to an agent. The agent and broker agreed that the broker would steal the coworker's desk drawer key the next morning, that the agent would make a copy of the key at lunch, and that the broker would then return the key to the coworker. They agreed that one of them would use the key to steal the watches sometime the next week. However, a waiter overheard their conversation and immediately reported it to the police. The next day, the broker stole the key and gave it to the agent, but they were both arrested before the agent could try to copy the key. Both are charged with conspiracy to commit larceny. The agent moves to dismiss the charge against him. The applicable jurisdiction has enacted a law that is identical to the Model Penal Code. Should the court grant the agent's motion to dismiss the conspiracy charge? A. No, because the broker's theft of the key is an overt act in furtherance of the conspiracy. B. No, because the conspiracy was complete upon the agreement between the broker and the agent. C. Yes, because the agent committed no overt act in furtherance of the conspiracy. D. Yes, because of the Wharton Rule.

A An overt act was not required at common law, but it is now a required element of a conspiracy under the MPC, federal law, and in a majority of states. When an overt act is required, the conspiracy crime is not complete until the overt act is performed in furtherance of the conspiracy. The overt act can be performed by any coconspirator, with or without the knowledge of all co-conspirators. Here, the broker's theft of the key is an overt act in furtherance of the conspiracy. Therefore, the agent is guilty of conspiracy to commit larceny. Answer choice B is incorrect because the MPC requires an overt act for a conspiracy to be complete. Answer choice C is incorrect because, although an overt act is required under the MPC, the overt act can be performed by any co-conspirator, with or without the knowledge of the other coconspirators. Answer choice D is incorrect. Under the Wharton Rule, if a crime requires two or more participants (e.g., adultery) there is no conspiracy unless more parties than are necessary to complete the crime agree to commit the crime. However, the Wharton Rule does not apply to conspiracies under the MPC because the MPC does not require 2 the participation of at least two conspirators. Therefore, because the jurisdiction has enacted law identical to the MPC, the Wharton Rule will not apply..

A defendant entered a convenience store wearing a Halloween mask and carrying a gun. He pointed the gun at the store clerk and told him to empty the cash register into a bag and hand it over. The clerk told the defendant that he needed a key from below the counter to unlock the cash drawer. He leaned over, pulled out a gun from behind the cash register, and quickly fired a shot. The defendant had seen the clerk reach for the gun, however, and jumped out of the way before the shot was fired. The bullet hit a customer, who later died from the injury. Is the defendant likely to be convicted of felony murder in a jurisdiction that follows the majority law? A. No, because the clerk could not be considered the agent of the defendant. B. No, because the defendant did not shoot the customer. C. Yes, because the customer's death was a natural and probable consequence of the robbery. D. Yes, because the defendant is strictly liable for any death that occurs during an inherently dangerous felony.

A Felony murder is an unintended killing proximately caused by and during the commission or attempted commission of an inherently dangerous felony, including a robbery. Most states apply the agency theory when a bystander is killed by a police officer or due to resistance by the victim of an inherently dangerous felony. Under this theory, the felon will not be liable for the death of a bystander caused by a felony victim or police officer because neither person is the felon's agent. In this case, the death of 2 the customer (a bystander) was caused by the clerk, who was not the agent of the defendant. Therefore, the defendant will not be liable for the customer's death. (B) is incorrect because, although the defendant did not shoot the customer himself, the defendant could still have been liable for the customer's death if an agent of the defendant, such as a co-felon, had killed the customer. (C) is incorrect because the proximate-cause theory, which holds that a bystander's death may fall within the felony murder rule because the death is a direct consequence of the felony, is not the majority rule. (D) is incorrect because a defendant is not strictly liable for any death that occurs during the commission of an inherently dangerous felony. Rather, the death must be a foreseeable result or a natural and probable consequence of the felony, caused by the defendant or a co-felon. Under the agency theory, a defendant is not liable for the death of a bystander caused by a felony victim, such as the clerk in this case.

A teenager walking down the sidewalk noticed that a car had been left unlocked with the engine running. The car's owner, a security guard, had parked the car beside the curb in front of his house in order to run in and get a pistol, which he was licensed to carry. As the owner came out his front door with the pistol, the teenager was opening the driver's door. The owner warned the teenager not to enter the car or he would shoot the teenager. The teenager ignored the warning. The owner shot and seriously injured the teenager. The owner has been charged with aggravated battery, which includes battery committed with a deadly weapon. Can the owner successfully assert defense of property as a justification for the shooting? (A) No, because the owner used deadly force. (B) No, because the owner was negligent in leaving the car running. (C) Yes, because the owner reasonably believed that the teenager was about to steal his car. (D) Yes, because the owner warned the teenager before using force against the teenager.

A Generally, deadly force may not be used in the defense of property. Here, the owner was not permitted to use deadly force to prevent the teenager from entering the car. Answer choice B is incorrect because the owner's right to use force to prevent the theft of his property is not lessened by his failure to take precautionary measures to prevent the theft. However, the owner's right to use force to protect his property generally does not extend to the use of deadly force. Answer choice C is incorrect. Although the property owner's belief that someone is stealing his property must be reasonable, the property owner generally may not use deadly force to prevent the theft of his property. Answer choice D is incorrect because a property owner generally is not justified in using deadly force to prevent the theft of his property, even though the property owner has warned the thief about the use of such force.

The defendant, over six feet tall and 250 pounds, became lost in an unfamiliar part of the city and did not have enough money for bus fare back to his house. He was scruffily dressed and appeared very unkempt. He approached a slightly built man who was standing alone at the bus stop and said in a gruff voice, "Do you have any money?" When the man replied, "Yes," the defendant said, "Give me five dollars!" The man reached into his pocket and handed the defendant a five-dollar bill. The man walked away from the bus and flagged down a passing patrol car, telling the police that he was just robbed. The defendant was arrested and charged with robbery. At the defendant's trial, he testified that he was not trying to rob anyone; he was just trying to beg for some money to get home. If the jury believes his testimony, the defendant should be found: A. Not guilty of robbery, because the defendant lacked the requisite criminal intent. B. Guilty of robbery, because the defendant took the man's money after intimidating him, and the effect of the defendant's size on the man, rather than the defendant's intent, is controlling. C. Not guilty of robbery, but guilty of assault, because the defendant placed the man in reasonable apprehension of bodily harm. D. Guilty of both robbery and assault, because assault is not a lesser included offense of robbery.

A If the jury believes the defendant's testimony, he did not have the intent for robbery. A robbery consists of (i) a taking (ii) of personal property of another, (iii) from the other's person or presence, (iv) by force or intimidation, (v) with the intent to permanently deprive him of it. Thus, robbery is basically an aggravated form of larceny in which the taking is accomplished by force or threats of force. If the jury believes the defendant's testimony that he was only trying to beg for some bus money, his only intent was to accept money that the man voluntarily gave him. The defendant did not have the requisite criminal intent because the defendant believed that the man was freely transferring his interest in the property to the defendant. Thus, the jury should find him not guilty of robbery. (B) is incorrect because the defendant's intent, rather than the effect of his conduct, is the controlling factor. Without any other facts, the defendant's conduct was sufficiently intimidating that a jury could conclude that he had an intent to rob. If the jury believes the defendant's testimony, however, it should find that he did not have the intent necessary for robbery. (C) is incorrect because the defendant's testimony, if believed, would also negate the intent necessary for assault. Criminal assault is either (i) an attempt to commit a battery, or (ii) the intentional creation of a reasonable apprehension in the mind of the victim of imminent bodily harm. If the defendant had intended to create an apprehension of imminent bodily harm, he would be liable for robbery because he would have been using that apprehension to permanently deprive the man of his property. However, assuming that the defendant was just trying to beg rather than rob, he will not be found to have intended to create the apprehension that he did. (D) is incorrect not only because the defendant's testimony disproves the requisite intent of either offense, but also because the assault would be a lesser included offense of the robbery under these circumstances. A lesser included offense is one that consists entirely of some (but not all) of the elements of the greater crime. Under the constitutional prohibition against double jeopardy, lesser included offenses "merge" into the greater offenses, in the sense that one may not be convicted of both the greater offense and a lesser included offense. Here, the robbery would be by intimidation: conduct by the defendant that was intended to create apprehension of imminent bodily harm in the man's mind. This element of the robbery offense would be enough to establish liability for assault as well, but robbery has additional elements; hence, the defendant could not have been found guilty of both robbery and the lesser included offense of assault.

A woman was moving out of state. As she was completing her packing one night, she realized that her neighbor had never returned a book that the woman had lent her. The woman walked over to her neighbor's house and knocked on the door. When she realized that no one was home, she let herself in through the unlocked garage, intending to take the book and return home. While she was in the neighbor's house, the woman remembered that the neighbor had a necklace that the woman had long admired. The woman decided to take the necklace, figuring that her neighbor probably would not notice it was missing until after the woman had moved. As the woman was heading toward the door, she heard her neighbor come in. The woman greeted her neighbor, and apologized for letting herself in to retrieve the book. The woman gave her neighbor a good-bye hug, and then left with the necklace in her pocket. The crimes below are listed ascending order of seriousness. What is the most serious crime for which the woman may be convicted? (A) Larceny. (B) Larceny by trick. (C) Robbery. (D) Burglary.

A Larceny is the trespassory taking and carrying away of the personal property of another with the intent to permanently deprive that person of the property. In this case, the elements of larceny are satisfied. The woman took and carried away her neighbor's necklace with the intent to keep it. Answer choice B is incorrect because a defendant is guilty of larceny by trick if she obtains possession (but not title) to property owned by another through fraud or deceit, with the intent to unlawfully convert it and later does so. In this case, the defendant did not obtain possession by fraud. Answer choice C is incorrect because the woman did not employ force or intimidation in taking the necklace, as required for robbery. Answer choice D is incorrect because the woman did not have the intent to commit a felony in the neighbor's house when she entered. Burglary is the breaking and entering of the dwelling of another at nighttime with specific intent to commit a felony (e.g., larceny, robbery, rape, murder) inside the dwelling. Because the woman did not have the specific intent to commit a crime when she entered the home, just to obtain a book that was rightfully hers, she is not guilty of burglary.

A career burglar and his friend planned to steal money from a retail store's safe by renting an apartment above the store and drilling through the floor to gain access to the store. They progressed with their drilling to the point that the only thing left to do was kick the floor to open the hole. However, on the evening that they were going to break into the store, the friend got cold feet. He told the burglar that he was not going through with the plan and left the apartment. The burglar decided to go through with the plan anyway, but he was caught by store security while he was trying to break open the safe. The jurisdiction follows the modern rules for attempt liability. If the burglar and the friend are charged with conspiracy and attempted larceny, what will be the likely result? (A) The burglar will be convicted of both conspiracy and attempted larceny, and the friend will be convicted of conspiracy only. (B) The burglar and the friend will be convicted of both conspiracy and attempted larceny. (C) The burglar and the friend will be convicted of attempted larceny only. (D) The burglar and the friend will be convicted of conspiracy only.

A The burglar and the friend are guilty of conspiracy. At common law, conspiracy was an agreement between two or more persons to commit an unlawful act or to commit a lawful act in an unlawful manner. Most states also require some overt act in furtherance of the conspiracy. Additionally, some jurisdictions recognize withdrawal as a defense to a conspiracy charge, but even those jurisdictions require that the co-conspirator somehow act to thwart the conspiracy. Withdrawal, however, does act as a defense to the subsequent crimes committed by the co-conspirators in furtherance of the conspiracy. When the burglar and the friend agreed to burglarize the store and began to carry out their plan, they committed common law conspiracy. Hence, (C) is incorrect. Given that the friend did nothing to thwart the conspiracy, his withdrawal provides no defense to the conspiracy charge, but it does immunize him from the subsequent crimes of the burglar. The burglar also committed an attempted larceny, but the friend did not. An attempt has two elements: (i) a specific intent to commit the target crime; and (ii) an overt act in furtherance of the crime. Under the modern M.P.C. approach, a complete and fully voluntary abandonment of the crime is a defense to an attempt charge. The abandonment must not be a temporary abandonment of the criminal purpose, nor may it be to find a different victim. In the instant case, both the burglar and the friend had the specific intent to commit the crime and performed overt acts in furtherance of the crime (e.g., drilling into the floor). Had they been caught at that time, both could be convicted of attempt. However, the friend voluntarily and fully abandoned the crime before completion. Thus, abandonment provides him with a defense to the attempt charge. Thus, (A) is correct and (B) is incorrect. (D) is incorrect because the burglar can be convicted of attempted larceny, as described above.

A father was terminally ill with a particularly painful form of cancer. His daughter visited him every evening in the hospital and for several months listened to his pleas to put him out of his misery. On her final visit, she gave her father a hug and then pulled a small revolver from her purse. She fired a shot at her father, killing him instantly. The daughter immediately broke into tears and surrendered to the police. The daughter was charged with her father's death. What is the most serious offense of which the daughter can be convicted? (A) First degree murder, defined by the jurisdiction as premeditated and deliberate killing of another human being. (B) Second degree murder, defined as any murder not classified as first degree murder. (C) Voluntary manslaughter. (D) No crime.

A The daughter can be convicted of first degree murder for her father's death. Therefore, (D) is incorrect. Murder is the unlawful killing of a human being with malice aforethought. In the absence of facts excusing the homicide or reducing it to voluntary manslaughter, malice aforethought exists if the defendant has (i) intent to kill, (ii) intent to inflict great bodily injury, (iii) awareness of an unjustifiably high risk to human life, or (iv) intent to commit a felony (felony murder doctrine). In this case, the daughter clearly had the intent to kill her father; her motive for doing so is irrelevant for establishing malice aforethought. It is also quite possible under the facts that the daughter acted with premeditation and deliberation, making her potentially liable for first degree murder. "Deliberate" means that the defendant reflected on the crime in a cool and dispassionate manner. "Premeditated" means that such reflection actually was undertaken, but it need only be for a very brief period. Here, the daughter could have reflected on the crime before she visited the hospital; the fact that she had a gun in her purse is circumstantial evidence of such reflection. Even if she did not make the decision to kill until moments before she pulled out the gun, she would still have been capable of premeditation and deliberation. (B) is therefore incorrect because it is not the most serious crime of which the daughter can be convicted. (C) is incorrect because of the absence of facts establishing adequate provocation that would reduce an intentional killing from murder to voluntary manslaughter. Adequate provocation is most frequently recognized in cases of (i) being subjected to a serious battery or threat of deadly force, and (ii) discovering one's spouse in bed with another person. While modern courts have broadened somewhat the scope of what constitutes provocation, the law does not presently recognize feelings of mercy toward the victim as an adequate provocation. Thus, the daughter can be convicted of murder for her father's death. (This question is a good illustration of why you must always treat the defendant objectively in your analysis of Criminal Law questions.)

The defendant is charged with the burglary of a home. Evidence presented at the defendant's trial indicates that he talked another person into assisting him. The jury is instructed on burglary, solicitation, conspiracy, and attempt. If the defendant is found by the jury to be guilty of burglary, which of the following is true? (A) He also may be found guilty of conspiracy, but not of solicitation or attempt. (B) He also may be found guilty of conspiracy and solicitation, but not of attempt. (C) He also may be found guilty of conspiracy or solicitation but not both, and he may not be found guilty of attempt. (D) He also may be found guilty of solicitation or attempt but not both, and he may not be found guilty of conspiracy.

A The defendant may also be found guilty of conspiracy if he is found guilty of burglary. One who solicits another to commit a crime cannot be convicted of both the solicitation and the completed crime. Likewise, one who completes a crime after attempting it may not be convicted of both the attempt and the completed crime. However, if conspirators are successful, they can be convicted of both criminal conspiracy and the crime they committed pursuant to the conspiracy (i.e., conspiracy does not merge with the completed offense). Thus, if the defendant is found guilty of burglary, he cannot also be convicted of either attempt or solicitation. The defendant can be convicted of conspiracy in addition to burglary (with conspiracy liability being based on the apparent agreement between the defendant and the other person to bring about the burglary of the home). (B) incorrectly states that the defendant may be found guilty of solicitation as well as the principal offense of burglary. (C) and (D) make the same incorrect assertion. In addition, (D) incorrectly states that the defendant may be found guilty of attempt as well as the completed crime, and that he may not be found guilty of conspiracy.

A professor had his laptop computer stolen from his office. He went to a computer resale shop to find a replacement and saw what he mistakenly thought was his computer. He questioned the owner of the shop, who told him that someone had just sold her the computer, but she refused to give him any information about the seller and refused to let him inspect the computer more closely. That night, after the shop was closed, the professor forced open the back door. After turning over the laptop to check the serial number, the professor realized that it was not the one that was stolen from his office. Nonetheless, he decided to keep it. The owner's clerk, who, unbeknownst to the professor, lived in a room in the back of the shop, heard someone breaking in and called the police. The professor was arrested as he was exiting the shop and is charged with larceny and burglary in a common law jurisdiction. Of which crimes may he be convicted and sentenced? A. Larceny, but not burglary. B. Burglary, but not larceny. C. Neither burglary nor larceny. D. Both burglary and larceny

A The defendant may be convicted of and sentenced for larceny only. At common law, burglary was the breaking and entering of the dwelling house of another at nighttime with the intent to commit a felony therein. The intent to commit a felony must exist at the time of the breaking and entering. Given that common law burglary requires the breaking and entering to be of the dwelling of another, an awareness of the building's use as a dwelling is a component of the mens rea for the crime. The professor's ignorance or mistake regarding the building's use as a dwelling negates the mens rea for that element of the crime of burglary. Furthermore, the professor did not have the intent to commit a felony at the time he broke into and entered the shop/dwelling. At that time, his only intent was to retrieve his own property, which would not have been sufficient for larceny (see below). For both of these reasons, the professor could not be convicted of and sentenced for burglary, thus making (B) and (D) incorrect answer choices. At common law, larceny was the taking and carrying away of the tangible personal property of another by trespass with the intent to permanently deprive the person of his interest in the property. In the instant case, the professor, at the time he broke into and entered the structure, did not have the intent to permanently deprive another of his interest of the laptop, given that he believed the laptop was the one stolen from his office. However, after turning over the laptop, the professor realized it was not his laptop, but he took it and carried it off anyway. This is sufficient for larceny. Thus, the professor may be convicted and sentenced for larceny, making (A) correct and (D) incorrect.

A state law defines the crime of aggravated battery as an act of force committed on a person who is physically or mentally disabled. A second statute provides that when a criminal statute does not designate a necessary mental state, the mental state required is criminal negligence. A 20-year-old male is charged with violating the aggravated battery statute for taunting and beating a mentally disabled student who was walking near his school for students with learning disabilities. At trial, the defendant testified that he was unaware of the statute and that he did not know the victim was mentally disabled. Furthermore, he testified that his only intent was to tease the victim. He stated that he picked up the victim and threw him to the ground just to see if he could do it. The victim testified that the defendant, who was much bigger in stature, started out by teasing him but then, after words were exchanged, picked him up and threw him to the ground causing bodily injury. Assume the jurisdiction follows the Model Penal Code analysis of fault. If the jury believes the evidence presented, may the defendant properly be convicted of aggravated battery? A. Yes, because the defendant failed to be aware of a substantial and unjustifiable risk that the victim was disabled. B. Yes, because the evidence is clear that the victim was mentally disabled. C. No, because the defendant was not consciously aware that the victim was mentally disabled. D. No, because the evidence shows that the defendant's intent was only to tease the victim

A The defendant may properly be convicted of aggravated battery because he committed an act of force on a mentally disabled person and failed to be aware of a substantial and unjustifiable risk that the victim was disabled. The state statute provides that when a criminal statute does not designate a necessary mental state, the mental state required is no more than criminal negligence. Under the Model Penal Code analysis of fault, a person acts negligently when he fails to be aware of a substantial and unjustifiable risk that circumstances exist or a result will follow. Here, the defendant should have been aware of the risk that the victim was disabled because the victim was walking near a school for students with learning disabilities. He therefore can be convicted of aggravated battery. (B) is incorrect because, although the evidence is clear the victim was disabled, the defendant can only be convicted if he had the mens rea 2 required by the statute—in this case, negligence. (C) is incorrect because the defendant did not need to be consciously aware that the victim was disabled; the statute only requires criminal negligence. (D) is incorrect because the defendant's intent to tease the victim is irrelevant. He picked up the victim and threw him to the ground, which constitutes an act of force under the statute.

The defendant and the victim got into a minor verbal altercation, concluding with the defendant lightly shoving the victim. The victim lost his balance and struck his head on the pavement, causing serious bodily injury. The defendant was charged with battery, which is defined in the jurisdiction as "purposely or knowingly causing serious bodily injury to another." Should the defendant be convicted of battery? (A) No, because the defendant did not know that the victim would be seriously injured. (B) No, because the defendant did not strike a serious blow to the victim. (C) Yes, because the defendant purposely shoved the victim. (D) Yes, because the victim suffered serious bodily injury.

A The defendant should not be convicted of battery. Under the statute's fault standards, a defendant must have acted purposely (i.e., with conscious intent to cause the result) or knowingly (i.e., with knowledge that his conduct will necessarily or very likely cause the result) as to the harmful result. The apparent inference to be drawn from the facts is that the defendant did not consciously desire, nor contemplate to a practical certainty, the serious injury to the victim that actually occurred. Had the defendant intended to cause such severe harm, he no doubt would have dealt the victim a strong blow rather than simply giving the victim a light shove. Therefore, as to the nature of the result, the defendant did not act with "purpose" or "knowledge" as those terms are defined in the Model Penal Code and modern criminal codes. (B), while close, is not as good an answer as (A) because it does not address the state of mind issue in the problem. A light shove might be sufficient for a battery as defined under a different set of facts (e.g., if the defendant believes that the victim would fall down stairs with a light shove). (C) is incorrect because it addresses the act but not the result. As defined in this question, battery must not only be committed by a purposeful act, but also be done with a "purposeful" or "knowing" state of mind as to the result. (D) is incorrect for much of the same reason—the state of mind requirement also applies to the result, as discussed above. The injury must have been purposely or knowingly caused, and that concept is not contained within choice (D).

An obsessive fan was heartbroken when her favorite actor announced his engagement to a well-known actress. The fan waited for the couple outside of a nightclub. When they arrived, the fan raised a gun and fired it at the actress, but as she fired, the actor's bodyguard spotted the gun and knocked it to one side. The bullet grazed the bodyguard's hand, causing minor injuries, and lodged in the actor's chest. Through prompt emergency medical treatment, the actor survived the shooting. Of whom may the fan be charged with attempted murder? (A) The actress. (B) The actor. (C) The actress and the actor. (D) The actress, the actor, and the bodyguard.

A The fan should be charged with the attempted murder of her original target, the actress. A criminal attempt is an act that, although done with the intention of committing a crime, falls short of completing the crime. The fan fired a gun at the actress; her intentional use of a deadly weapon permits an inference that she had the intent to kill the actress. If her plan had succeeded, she would have been guilty of murder. (B), (C), and (D) are incorrect because the fan did not have the intent to kill the actor. Had the fan actually killed the actor, her intent to kill the actress could have been transferred to make her guilty of murdering the actor, but the doctrine of transferred intent does not apply to an attempt. (D) is also incorrect because the fan did not have intent to kill the bodyguard.

The defendant is charged with the burglary of a home. Evidence presented at the defendant's trial indicates that he talked another person into assisting him. The jury is instructed on burglary, solicitation, conspiracy, and attempt. If the defendant is found by the jury to be guilty of burglary, which of the following is true? A. He also may be found guilty of conspiracy, but not of solicitation or attempt. B. He also may be found guilty of conspiracy and solicitation, but not of attempt. C. He also may be found guilty of conspiracy or solicitation but not both, and he may not be found guilty of attempt. D. He also may be found guilty of solicitation or attempt but not both, and he may not be found guilty of conspiracy.

A The father lacked the specific intent to kill that is required for attempted murder. However, the circumstances surrounding the "incendiary device" constitute factual impossibility and will not afford the father a defense to attempted arson. Criminal attempt is an act that, although done with the intention of committing a crime, falls short of completing that crime. To be guilty of attempt, the defendant must have the intent to perform an act and obtain a result that, if achieved, would constitute a crime. Regardless of the intent that would suffice for the completed offense, attempt always requires a specific intent to commit the target offense. Also, the defendant must have committed an act beyond mere preparation for the offense. Here, to be guilty of attempted murder, the father must have had the specific intent to kill his son's coach, even though the intent to inflict great bodily injury would be sufficient mens rea for murder. However, the facts indicate that the father intended at most only to injure the coach rather than kill him. Thus, the father cannot be guilty of attempted murder. However, the father did intend to burn the coach's home; therefore, he had the specific intent to commit arson by means of placing an incendiary device on the coach's porch, and his placing the device was an act beyond mere preparation for this crime. Although the device could not have actually burned the coach's house, it is no defense to attempt that it would have been impossible for the defendant to complete his plan. This is factual impossibility and is not a defense. Thus, the father is guilty of attempted arson.

Two robbers planned to commit armed robberies targeting older victims. However, when the time came to actually commit the robbery, one of the robbers, thinking that the potential victim looked too much like his grandmother, backed out and told his cohort that he was going home. The second robber went ahead with the plan and robbed the elderly victim, who died of a heart attack due to the stress of the robbery. The second robber was arrested and implicated the first robber. What crime(s) has the first robber committed? A. No crimes. B. Conspiracy. C. Murder. D. Murder and conspiracy.

A The first robber is guilty of conspiracy but not murder. Conspiracy consists of: (i) an agreement between two or more persons; (ii) an intent to enter into an agreement; and (iii) an intent to achieve the objective of the agreement. In addition, most states require an overt act in furtherance of the conspiracy, although an act of mere preparation will usually suffice. The conspiracy was complete when the robbers agreed to commit the robbery and targeted their first victim. The first robber's withdrawal is no defense to the conspiracy charge because a conspiracy is complete as soon as an agreement is made and an overt act is committed. Hence, (A) is incorrect. The first robber is not guilty of murder, however, because of his withdrawal. The murder charge would be based on felony murder, because the second robber caused the foreseeable death of the elderly victim from the heart attack during the commission of the felony. However, a conspirator may limit his liability for subsequent acts of other conspirators by performing an affirmative act that notifies the other members of the conspiracy in time for the other members to have the opportunity to abandon their plans. Here, the first robber told the second robber that he was going home in time for the second robber to abandon his plans. Hence, he is not liable for felony murder arising from the robbery, making (C) and (D) incorrect.

The defendant wanted to steal some papers from an accountant's office safe, and so he arranged to have a meeting with the accountant at the office. When the accountant left the room, the defendant put a knockout drug that he had prepared in the accountant's coffee. After the accountant passed out from drinking the coffee, the defendant stole the papers. Unfortunately, the defendant miscalculated the dosage, and the accountant died. What is the most serious offense of which the defendant can be convicted? A. Murder. B. Voluntary manslaughter. C. Involuntary manslaughter. D. None of the above.

A The most serious offense of which the defendant can be convicted is murder. Murder at common law is the unlawful killing of a human being with malice aforethought. Malice aforethought exists if the defendant has any of the following states of mind: (i) intent to kill (express malice); (ii) intent to inflict great bodily injury; (iii) reckless indifference to an unjustifiably high risk to human life ("abandoned and malignant heart"); or (iv) intent to commit a felony (limited in most states to inherently dangerous felonies). Because malice will be implied from the intent to commit the underlying felony, even an accidental killing committed during the course of a felony is murder. Here, the defendant intended to, and did in fact, commit a dangerous felony, robbery, which is the taking of personal property of another from the other's person or presence, by force or intimidation, with the intent to permanently deprive him of it. The knockout drops constituted force, sufficient to overcome the accountant's resistance; hence, the defendant has committed a robbery and his intent constitutes implied malice 2 for purposes of felony murder. Therefore, (A) is correct and (B), (C), and (D) are incorrect.

A restaurant patron looked up from her dinner and saw that another woman had taken her coat and was walking out the door with it. The patron quickly left and followed the woman, and saw the woman enter a nearby house. Later, when the patron knew that nobody was in the house, she opened a window to the house, climbed in, and got what she thought was her coat out of the front closet. When she put the coat on, however, she noticed for the first time that the coat was not her coat. But, because it was so cold outside, she decided to wear this coat home and to return it the next day. The following day she changed her mind and decided to keep the coat. Of what crime is the patron guilty? A. Larceny. B. Burglary. C. Both larceny and burglary. D. Neither larceny nor burglary.

A The patron is guilty of larceny, but not burglary. When the patron took the coat, knowing it was not her own, she committed a trespassory taking. However, the 4 facts state that she did not intend to permanently (or for an unreasonably long time) deprive the other woman of her possessory interest in the coat. Thus, at that time, she did not have the intent to commit larceny. However, under the continuing trespass doctrine, if a defendant takes property with a wrongful state of mind, but without the intent to commit larceny, and later, while still in possession of the property, forms the intent to steal it, the trespass involved in the initial wrongful taking is regarded as "continuing," and the defendant is guilty of larceny. Here, the patron's initial taking of the coat was wrong, in that she knew the coat did not belong to her when she borrowed it without permission. Thus, when she later formed the intent to permanently keep the coat, she committed larceny under the continuing trespass doctrine. However, she did not commit burglary. Burglary is the breaking and entering into the dwelling of another at nighttime with the intent to commit a felony therein. The intent to commit a felony must exist at the time of the breaking and entering. In the instant case, when the patron broke and entered into the woman's home, the only intent she had was to retrieve her own property, which would not be a felony. Without the intent to commit a felony at the time of the breaking and entering, the patron would not be guilty of burglary. Thus, (A) is correct, and (B), (C), and (D) are wrong.

A woman decided to steal a necklace that belonged to her neighbor. She knew where the neighbor kept the necklace because she had been in the neighbor's house on many occasions when the neighbor had taken off the necklace and put it away in a jewelry box in the bathroom. One night, the woman went to the neighbor's house. The neighbor was away and the house was dark. The woman opened the bathroom window, saw the jewelry box on the counter, and started to climb inside. As her leg cleared the window sill, the neighbor's cat let out a loud screech. Terrified, the woman bolted back outside and fled. The crimes below are listed in descending order of seriousness. What is the most serious crime committed by the woman? (A) Burglary. (B) Attempted burglary. (C) Attempted larceny. (D) No crime.

A The woman is guilty of burglary because she unlawfully entered the neighbor's house at night with intent to commit a felony (larceny). The woman's actions constituted the requisite "entry" of the neighbor's house. See Wayne R. LaFave, Substantive Criminal Law § 21.1 (2d ed. 2003) (To constitute burglary it is "sufficient if any part of the actor's person intruded, even momentarily, into the structure. Thus it has been held that the intrusion of a part of a hand in opening a window, or the momentary intrusion of part of a foot in kicking out a window, constituted the requisite entry."). (B) is incorrect because the woman's action proceeded beyond the point of attempted burglary. The woman is guilty of burglary because she unlawfully entered the neighbor's house at night with intent to commit a felony (larceny). (C) is incorrect because the woman may have been guilty of attempted larceny, but that crime arguably would merge into, and in any event was less serious than, the burglary crime. (D) is incorrect because the woman is guilty of burglary because she unlawfully entered the neighbor's house at night with intent to commit a felony (larceny).

The same scenario, except that D is the only person in the area and does not swim to V's aid. Does actus reus exist? A. Yes. B. No.

B

A drug addict entered a pawnshop with a starter pistol that could not fire real bullets, intending to rob it so he could buy drugs. However, once inside the pawnshop, the addict was too afraid to do anything and turned to leave. He bumped into another customer who was coming in as he was going out, and the pistol fell from his coat pocket. The shopkeeper saw it and realized what the addict had intended. She chased after him and flagged down a patrol car, and he was arrested a block away. The addict is charged with burglary, which is defined in this jurisdiction as "breaking and entering of any building for the purpose of committing a felony." His best defense would be which of the following? A. His pistol was inoperative. B. The pawnshop was open for business. C. He had abandoned his plan to commit the robbery. D. As a drug addict, he was incapable of forming the requisite intent.

B Because this statute requires that the defendant "break and enter," the addict's best defense is that, because the pawnshop was open to the public, he had not "broken" in but entered with the implied consent of the pawnshop owner. (A) is wrong because it is immaterial to the crime of robbery (the underlying felony in this burglary) that the robber was incapable of carrying out his threat of harm, as long as the victim reasonably believed that he was. (C) is not the best answer because the statute requires only that the defendant enter the building with the intent to commit a felony. The fact that the intent was abandoned after entry would not make the defendant not guilty of burglary. (D) is wrong because there is nothing in the facts indicating that the addict was incapable of realizing what he was doing, and being a drug addict in and of itself does not show that the defendant is incapable of forming an intent to commit a crime.

One night, when no one was home, an individual broke into a residence to steal a painting by a particular artist. Not finding the painting, the individual instead stole a drawing by the same artist. Can the individual properly be charged with burglary? A. Yes, because the individual stole the drawing while in the residence. B. Yes, because the individual, when entering the residence, intended to steal the painting. C. No, because the individual did not steal the painting. D. No, because the individual did not use force or intimidation to steal the drawing.

B Burglary is the (i) breaking and (ii) entering (iii) of the dwelling (iv) of another (v) at nighttime (vi) with the specific intent to commit a felony therein. Larceny is the (i) trespassory (ii) taking and carrying away (iii) of the personal property of another (iv) with the intent to permanently deprive that person of the property (i.e., intent to steal). Here, the individual had the intent to commit larceny at the time the individual broke into the residence. Consequently, the individual can properly be charged with burglary. (A) is incorrect because the crime of burglary is complete when a defendant breaks and enters a dwelling at night with the intent to commit a felony therein. Therefore, the defendant's subsequent commission of larceny with regard to a different item of personal property is irrelevant with regard to the burglary charge. (C) is incorrect because the crime of burglary is complete when a defendant breaks and enters a dwelling at night with the intent to commit a felony therein. Therefore, it is no defense that the defendant is unable to successfully commit the intended felony. (D) is incorrect because, although robbery requires the use of force or intimidation in the commission of larceny, burglary does not.

In which of the following circumstances is the defendant most likely to be convicted of common law murder? A. While at a bar, a patron throws a drink in the defendant's face, and the defendant, in a rage, stabs and kills the patron. B. The defendant is hunting deer and shoots and kills another hunter. C. The defendant drives his car up onto and along a crowded city sidewalk, killing a pedestrian. D. During an argument, the defendant shoves his opponent, who stumbles backward several feet and falls through a large retail display window. Glass shards penetrate the opponent's heart, killing him.

C Defendant is most likely to be convicted of murder for driving his car onto the sidewalk. Murder at common law is the unlawful killing of a human being with malice aforethought. Malice aforethought exists if the defendant has any of the following states of mind: (i) intent to kill (express malice); (ii) intent to inflict great bodily injury; (iii) reckless indifference to an unjustifiably high risk to human life ("abandoned and malignant heart"); or (iv) intent to commit a felony. In the case of (ii), (iii), and (iv), the malice is "implied." In this type of question, you must examine each of the scenarios to determine the correct answer. In (C), driving onto and along a crowded sidewalk is the kind of reckless conduct likely to cause serious bodily injury, which justifies a "reckless indifference" theory of common law murder. (A) is not correct because the battery on the defendant might be considered a provoking event, which would mitigate the homicide to voluntary manslaughter, which is an intentional killing distinguishable from murder by the existence of adequate provocation; i.e., a killing in the heat of passion. (B) is incorrect because there is no indication of intent or reckless disregard of life that would be required for murder. (D) similarly appears to be an accidental death without any indication of malice aforethought.

One night, when no one was home, an individual broke into a residence to steal a painting by a particular artist. Not finding the painting, the individual instead stole a drawing by the same artist. Can the individual properly be charged with burglary? (A) Yes, because the individual stole the drawing while in the residence. (B) Yes, because the individual, when entering the residence, intended to steal the painting. (C) No, because the individual did not steal the painting. (D) No, because the individual did not use force or intimidation to steal the drawing.

B Burglary is the (i) breaking and (ii) entering (iii) of the dwelling (iv) of another (v) at nighttime (vi) with the specific intent to commit a felony therein. Larceny is the (i) trespassory (ii) taking and carrying away (iii) of the personal property of another (iv) with the intent to permanently deprive that person of the property (i.e., intent to steal). Here, the individual had the intent to commit larceny at the time the individual broke into the residence. Consequently, the individual can properly be charged with burglary. Answer choice A is incorrect. Because the crime of burglary is complete when a defendant breaks and enters a dwelling at night with the intent to commit a felony therein, the defendant's subsequent commission of larceny with regard to a different item of personal property is irrelevant with regard to the burglary charge. Answer choice C is incorrect. Because the crime of burglary is complete when a defendant breaks and enters a dwelling at night with the intent to commit a felony therein, it is no defense that the defendant is unable to successfully commit the intended felony. Answer choice D is incorrect because, although robbery requires the use of force or intimidation in the commission of larceny, burglary does not.

After drinking heavily at his bachelor party at a beachfront resort, the groom was helped into a speedboat by a few of his friends and transported to a small island off the coast as a joke. They left him on the island, which had a small shelter but no communication facilities, without telling anyone else. As a result, the groom missed his wedding the next day. One of the participants was charged with kidnapping, which is defined in the jurisdiction as the unlawful movement or concealment of a person without his consent. In his defense, the participant claims that he was so intoxicated that he did not realize what he was doing, and that the groom had consented to being left on the island. Which of the following would NOT be helpful to his defense? (A) The groom was not legally intoxicated that evening. (B) Kidnapping is a general intent crime in the jurisdiction. (C) Kidnapping is a specific intent crime in the jurisdiction. (D) The participant had overheard the groom say that he was not sure about going through with the wedding.

B It would be least helpful to a kidnapping participant's defense if kidnapping were a general intent crime in the jurisdiction. Although courts have not always clearly defined "general intent," the mental state required for the material elements of the offense are analogous to "recklessness" under the Model Penal Code: conscious disregard of a substantial or unjustifiable risk that the material element exists or will result from his conduct. Thus, the defendant need not be certain that his conduct will cause the result or that the attendant circumstances required by the crime exist; it is enough if he is aware of a high likelihood of that result or circumstance. In contrast, a specific intent crime requires the doing of an act with a conscious intent or objective. Most importantly for the participant's purposes, defenses such as voluntary intoxication and unreasonable mistake of fact are not recognized as defenses to general intent crimes, but are for specific intent crimes. If the jurisdiction treats kidnapping as a specific intent crime, the participant's intoxication could be used to show that he was incapable of forming the requisite intent or that he mistakenly believed that the groom had consented to being left on the island. For specific intent crimes, any mistake of fact, even if unreasonable, is a defense. In contrast, voluntary intoxication is not a defense to a general intent crime, and any mistake of fact offered to negate a general intent must be reasonable to be valid. Hence, it would be helpful to the participant's defense if the jurisdiction treated the offense as a specific intent crime, but not if it were treated as a general intent crime. Thus, (B) is correct because it is not helpful, and (C) is incorrect because it is helpful. (D) is incorrect because that fact may be helpful to the participant's defense. If he believed that the groom wanted to be left on the island, he may not have had the intent required for the offense. (A) is incorrect because it is helpful to the participant's defense. The offense is defined as the unlawful movement or concealment of a person without his consent. If the participant was not legally intoxicated, his consent would be a valid defense; if he was legally intoxicated, it could be argued that he was incapable of consenting, thus negating the participant's defense.

A dog sitter customarily went into her clients' homes, with keys they provided to her, to care for their pets. Over the years, she grew to especially love one particular dog. She would regularly enter the home of her client on days she was not scheduled to watch the dog and would take the dog to the park. One afternoon, the dog she loved wandered into her yard as she was doing yard work. She did not immediately call her client, but instead decided to keep the dog for a few days. A week later when she returned him to the client, the client stated that she so enjoyed having the house to herself that she was going to sell him. The dog sitter immediately purchased him and brought him back to her home. Were the dog sitter's actions sufficient to meet the requirements of larceny? (A) No, because she had a key to enter the home. (B) No, because she was only going to keep the dog for a few days. (C) Yes, because she kept the dog despite knowing who owned it. (D) Yes, because she would enter the client's home regularly and take the dog to the park.

B Larceny is the trespassory taking and carrying away of the personal property of another with the intent to permanently deprive that person of the property (i.e., intent to steal). The property must be taken without the owner's consent. Although the dog sitter did retain the property of another with the intent to deprive the owner of the dog temporarily, she never intended to keep him permanently until the client sold him to her. Answer choice A is incorrect because, although she 10 had permission to enter the home and watch the dog, that would not give her the authority to permanently deprive the client of the dog. Answer choice C is incorrect because she never had the intent to permanently deprive her client of the dog. While it is possible for a larceny to be committed even if the intent to steal is not formed until after the taking under the continuing trespass rule, here, the dog sitter did not intend to keep the dog permanently until after she was given permission to do so. Therefore, since the dog's owner consented to the dog sitter keeping the dog, there was no continuing trespass. Answer choice D is incorrect because the act, while problematic, was not larceny. She was certainly trespassing, and could likely be found guilty of some crime, as the key was given only to enter when she was dog-sitting, but she did not intend to permanently deprive the owner of the dog. Accordingly, she could not be found guilty of larceny.

A woman discovered that her boyfriend was cheating on her. She approached the bartender at her local bar, who she had heard was a member of a biker gang, and offered him $500 to beat up her boyfriend. The bartender agreed immediately. The bartender was actually an undercover officer, however, and he did not intend to beat up the boyfriend. The woman was arrested the next day and charged with solicitation, conspiracy, and assault. The jurisdiction recognizes the common law regarding these crimes. Are the charges against the woman proper? A. Yes as to solicitation, but no as to assault and conspiracy. B. Yes as to solicitation and assault, but no as to conspiracy. C. Yes as to solicitation and conspiracy, but no as to assault. D. No as to all charges.

B Solicitation is the encouragement of another person to commit a crime with the intent that the other person commit the crime. The woman was guilty of solicitation when she encouraged the bartender to beat up her boyfriend. It is irrelevant that the bartender never intended to commit the crime. Assault is an attempt to commit a battery (the unlawful application of force to the person of another, which can be applied through a third person acting under the defendant's direction). Attempt requires a substantial step toward the commission of a crime, such as solicitation of an innocent agent to engage in criminal conduct, coupled with intent to commit that crime. Both elements are satisfied in this case. Thus, the woman may be charged with both solicitation and assault. Answer choice A is incorrect because the elements of assault are satisfied. Answer choices C is incorrect because there was no agreement, and thus no conspiracy. Conspiracy requires an agreement between two or more persons. When only one party has the intent to agree, there is no conspiracy. In this case, the bartender never intended to agree, and thus there was no conspiracy. Answer choice D is incorrect because the woman can be properly charged with both solicitation and assault.

A woman searched on the internet for a hit man who would kill her husband in exchange for cash. She found a purported mercenary in a chat room who agreed to kill the husband in return for $50,000. The mercenary turned out to be an undercover police officer and the woman was arrested. With which of the following crimes could the woman properly be charged? A. Conspiracy only. B. Solicitation only. C. Both conspiracy and solicitation. D. Neither conspiracy nor solicitation.

B Solicitation is the enticing, encouraging, requesting, or commanding of another person to commit a crime with the intent that the other person commits the crime. Here, the woman enticed the undercover police officer to commit the murder, with the intent that he would do so. Factual impossibility is not a defense to solicitation. Accordingly, she could properly be charged with solicitation. Answer choice A is incorrect. At common law, to be guilty for a conspiracy, there must have been an agreement between two parties. When only one conspirator has the intent to agree, such as when the other conspirator is a governmental agent there is no conspiracy, unless another participant is involved. Here, only one of the two parties involved had a true intent to agree to the crime. Accordingly, conspiracy could not properly be charged. Answer choice C is incorrect, as the woman could not properly be charged with conspiracy, as noted above. Answer choice D is incorrect, as the woman could properly be charged with solicitation, as noted above..

It was a dark and rainy night. Out of the gloomy darkness, the accused approached the victim from behind. She stuck a gun in the victim's back and shouted, "Your money or your life!" The victim was scared out of his wits and collapsed in a faint on the spot. The accused lifted the victim's wallet and stuck it in the back pocket of her coveralls. The accused took off at a trot, but after she had traveled about 10 feet from the victim, the wallet slipped out of her pocket and fell to the ground. The accused did not realize this until she arrived home and found the wallet missing. In the meantime, the victim revived; he found the wallet with all its contents intact. The victim called the police to report the crime and later identified the accused in a lineup at the police station. The accused should be convicted of: (A) Larceny only. (B) Robbery only. (C) Larceny and robbery. (D) Nothing.

B The accused should be convicted of robbery but cannot also be convicted of larceny because larceny is a lesser included offense of robbery. Robbery consists of a taking of the personal property of another from the other's person or presence, by force or intimidation, with the intent to permanently deprive him of it. The accused took the victim's personal property (his wallet) from his person at gunpoint, intending to permanently deprive him of his property. Although the victim fainted, this taking was accomplished by force, because the accused's threat of shooting the victim precipitated his fainting. Thus, the accused can be convicted of robbery. Larceny consists of a taking and carrying away of the tangible personal property of another by trespass, with intent to permanently deprive the person of his interest in the property. Although the accused carried the victim's wallet only 10 feet before it slipped out of her pocket, this is a sufficient carrying away for purposes of larceny. Thus, the accused could be convicted of larceny. However, one may not be convicted of both a greater offense and a lesser included offense. A lesser included offense is one that consists entirely of some, but not all, elements of the greater crime. Larceny is a lesser included offense of robbery because larceny consists of all the elements of robbery except for force or intimidation. Indeed, robbery can be considered an aggravated form of larceny, in which the taking is accomplished by force or threat of force. Thus, the accused cannot be convicted of both larceny and robbery. Therefore, (C), which would allow conviction of both crimes, is wrong. (A) is wrong because it would preclude a conviction 4 for robbery and, as has been explained, the accused can be convicted of robbery. (D) is wrong because the accused can be convicted of robbery.

A woman attempted to charge a coat using a credit card that she knew her friend had stolen. The store's electronic credit reporting system indicated the account was defunct, and the store clerk refused to complete the transaction. The woman left the store and held up a couple at gunpoint in a nearby park. However, the couple had just been robbed of all their valuables and was heading over to the police station to report the crime, so the woman got nothing. The woman was arrested and charged with attempting to obtain property by false pretenses and attempted robbery. The court should find the woman: A. Not guilty of attempting to obtain property by false pretenses and not guilty of attempted robbery. B. Guilty of attempting to obtain property by false pretenses and guilty of attempted robbery. C. Not guilty of attempting to obtain property by false pretenses and guilty of attempted robbery. D. Guilty of attempting to obtain property by false pretenses and not guilty of attempted robbery

B The court should find the woman guilty of both crimes. To convict for attempt, the prosecutor must establish that the defendant (i) had the specific intent to commit the crime; and (ii) engaged in behavior that constituted a substantial step toward commission of the crime and was strongly corroborative of the defendant's criminal purpose. The woman could be found guilty of attempting to obtain property by false pretenses. She had the specific intent to commit the crime, and the jury could find she took a substantial step toward commission of it. Thus, (A) and (C) are incorrect. Similarly, the woman had the specific intent to commit the robbery and took a substantial step toward commission of the crime. It is no defense to attempt that it was factually impossible for the defendant to complete the plan; she did all the things she intended to do. Thus, (D) is incorrect and (B) is correct.

A beautiful woman preyed upon lonely, desperate men looking for love on the Internet by getting a potential suitor to fall in love with her and then propose marriage to her. After a suitor would propose to her and give her an engagement ring, the woman would have a replica of the engagement ring made at a fraction of the cost of the real one. Shortly thereafter, the woman would break off the engagement, return the knock-off version of the engagement ring to the duped suitor, and then sell the real engagement ring for a sizeable profit. After scamming her most recent suitor, the suitor tried to return the knock-off ring to a high-end jewelry store the same day that he was dumped. The jewelry store marked all of their rings with a very specific insignia that could not be replicated. Once they realized that the ring was a knock-off, they called the police and the woman was arrested. If the woman is guilty of any crime, she is most likely guilty of: (A) Embezzlement. (B) False pretenses. (C) Larceny. (D) Larceny by trick.

B The crime of false pretenses is a false representation of a past or present material fact made by the defendant with the intent to cause the victim to pass title to the defendant, and title is passed. False pretenses requires obtaining title to the property of another person through the reliance of that person on a known false representation of a material past or present fact, and the representation is made with the intent to defraud. Here, the suitor relied on the false representation that the woman loved him and intended to marry him, and the woman made this representation with the intent to defraud the suitor and obtain title to the engagement ring. Answer choice A is incorrect. Embezzlement is the fraudulent conversion of the property of another by a person who is in lawful possession of the property. The woman had possession of the engagement ring because the suitor gave her the ring as a symbol of their agreement to marry. Additionally, the woman had the intent to defraud the suitor and convert the property because she returned a knock-off version of the engagement ring and sold the real one. However, because she obtained title to the ring through a false representation-that she loved the suitor and intended to marry him, the woman will more likely be convicted of false pretenses. Answer choice C is incorrect. Larceny is the trespassory taking and carrying away of the personal property of another 9 (without his consent) with the specific intent to permanently deprive the owner of the property at the time of the taking. Here, there was no trespassory taking and caring away of the engagement ring from the suitor without his consent, so there was no larceny. Answer choice D is incorrect. Larceny by trick occurs when the defendant fraudulently induces the victim to deliver possession of the property to the defendant. Here, the woman obtained title to the engagement ring, not just possession, so she is most likely guilty of false pretenses.

A customer approached a clerk at a local mini-mart and offered to exchange a pair of sunglasses for $10 worth of gas. The clerk refused, saying he could only accept cash. The customer then pulled a knife out of his pocket and told the clerk he wanted a fill-up. The clerk, who was quite a bit older than the customer, gave him some "fatherly advice" that crime does not pay. In response to the advice, the customer put the knife away. Feeling sorry for the customer, the clerk then agreed to give him $10 worth of gas for the sunglasses. The customer got the gas and then left. The clerk then discovered that the customer had taken the sunglasses from a display case in the store and clipped the tag off before offering them to the clerk. The customer was apprehended shortly thereafter. Which of the following statements is correct regarding the customer's conduct? (A) The customer can be convicted of larceny by trick and attempted robbery. (B) The customer can be convicted of false pretenses and attempted robbery. (C) The customer can be convicted of larceny by trick but not attempted robbery because he voluntarily abandoned the attempt. (D) The customer can be convicted of false pretenses but not attempted robbery because he voluntarily abandoned the attempt.

B The customer can be convicted of false pretenses and attempted robbery. False pretenses consists of obtaining title to the property of another by an intentional or knowing false statement of past or existing fact with intent to defraud another. With regard to the false representation, all that is required is that the defendant create a false impression as to a matter of fact, which is what happened here. The customer obtained title to the gasoline by creating a false impression that he owned the sunglasses that he was offering in exchange, and he had the requisite intent to be convicted of false pretenses. He can also be convicted of attempted robbery because he attempted a taking of the property of another in the presence of the victim by force and with the intent to permanently deprive the victim of it. The fact that he was persuaded not to carry out the robbery does not affect his liability for attempt; that crime was completed as soon as he pulled out a knife and demanded the gas. (A) and (C) are incorrect because larceny by trick occurs when possession of the property is obtained by the defendant's misrepresentations, whereas false pretenses is the appropriate offense when the misrepresentations have prompted the victim to convey title to the property to the defendant. Here, the clerk intended to convey title to the gas to the customer in exchange for the sunglasses. (C) and (D) are incorrect because the majority rule is that abandonment is not a defense to attempt. As discussed above, the crime of attempted robbery was completed as soon as the customer pulled the knife out of his pocket and demanded the gas.

The defendant approached a clerk at a local gas station/mini-mart and offered to exchange a pair of sunglasses for some gas. The clerk refused, saying he could only accept cash. The defendant then pulled a knife out of his pocket and told the clerk he wanted a fill up. The clerk, who was quite a bit older than the defendant, gave him some "fatherly advice" that crime does not pay. In response to the advice, the defendant put the knife away. Feeling sorry for the defendant, the clerk then agreed to give him some gas for the sunglasses. The defendant got the gas and then left. The clerk then discovered that the defendant had taken the sunglasses from a display case in the store and clipped the tag off before offering them to the clerk. The defendant was apprehended shortly thereafter. Which of the following statements is correct regarding the defendant's conduct? (A) The defendant can be convicted of larceny by trick and attempted robbery. (B) The defendant can be convicted of false pretenses and attempted robbery. (C) The defendant can be convicted of larceny by trick but not attempted robbery because he voluntarily abandoned the attempt. (D) The defendant can be convicted of false pretenses but not attempted robbery because he voluntarily abandoned the attempt.

B The defendant can be convicted of false pretenses and attempted robbery. False pretenses consists of obtaining title to the property of another by an intentional or knowing false statement of past or existing fact with intent to defraud another. With regard to the false representation, all that is required is that the defendant create a false impression as to a matter of fact, which is what happened here. In this case, the defendant obtained title to the gasoline by creating a false impression that he owned the sunglasses that he was offering in exchange, and he had the requisite intent to be convicted of false pretenses. He can also be convicted of attempted robbery because he attempted a taking of the property of another in the presence of the victim by force and with the intent to permanently deprive the victim of it. The fact that he was persuaded not to carry out the robbery does not affect his liability for attempt; that crime was completed as soon as he pulled out a knife and demanded the gas. (A) and (C) are incorrect because larceny by trick occurs when possession of the property is obtained by the defendant's misrepresentations, whereas false pretenses is the appropriate offense when the misrepresentations have prompted the victim to convey title to the property to the defendant. Here, the clerk intended to convey title to the gas to the defendant in exchange for the sunglasses. (C) and (D) are incorrect because the majority rule is that abandonment is not a defense to attempt. As discussed above, the crime of attempted robbery was completed as soon as the defendant pulled the knife out of his pocket and demanded the gas.

A defendant was owed $500 by his bookie. The defendant went to the bookie's house one night to get his money, but the bookie was not at home. So, the defendant opened an unlocked window and entered the house to see if he could find the $500 due him. He could not find the cash, so he decided to take a painting that he knew was worth substantially more than $500. He later sold it for $1,000 and kept the proceeds. The defendant is charged with burglary. The jurisdiction in which the bookie's house is located retains the common law requirements for burglary. Should the defendant be found guilty? A. No, because there was no breaking, since he did not use force to enter the house. B. No, because he did not have the intent to commit a felony when he entered the house. C. Yes, because he did have the intent to commit a felony when he entered the house. D. Yes, because the painting was worth substantially more than $500.

B The defendant will not be guilty of burglary because he did not have the intent to commit a felony at the time he entered the bookie's house. At common law, the elements of burglary are (i) a breaking (ii) and entry (iii) of a dwelling (iv) of another (v) at nighttime (vi) with the intent of committing a felony therein. Here, the defendant entered the bookie's house with the intent only to obtain repayment of a debt, which does not satisfy the intent required for larceny. Given that the defendant believed he was entitled to take the money as repayment of the debt, he did not intend to permanently deprive the bookie of his property. The fact that he later decided to steal the painting will not establish the requisite intent; it must exist at the time of the breaking and entering. Thus, both (C) and (D) are wrong. (A) is wrong because, although actual breaking requires some use of force to gain entry, minimal force is sufficient, as when the burglar opens an unlocked window.

The owner of a garage and one of his mechanics had a dispute over the amount of wages due for work that the mechanic had done on cars. The owner alleged that he paid the mechanic for work that the mechanic did not perform, while the mechanic contended that she did the work. After arguing over the matter for over an hour, the mechanic quit. The owner did not allow the mechanic to take her tools with her, stating that he was keeping her tools until she repaid the money that he had overpaid her. The mechanic met with a friend and discussed the situation with him. After hearing what had happened, the friend, believing that the owner was unlawfully retaining the mechanic's tools, suggested that he could pose as a garage client and retrieve the tools for the mechanic, and the mechanic agreed. The plan succeeded, except for the fact that the friend took tools that belonged to a new employee. The owner immediately realized what had happened, and he gave the friend's license number to the police. The friend was arrested before he could show the tools to the mechanic. Of the following, which is the best argument for the friend as a defense to a charge of larceny in a common law jurisdiction? A. He had the consent of the tools' owner. B. He thought the tools belonged to the mechanic. C. He intended to return the tools to the mechanic rather than keep them. D. He was apprehended by the police before the mechanic could inspect the tools.

B The friend's best argument is that he thought the tools belonged to the mechanic. Larceny is the taking and carrying away of the tangible personal property of another by trespass with the intent to permanently (or for an unreasonable time) deprive the owner (or person in possession) of his interest in the property. Thus, larceny is a specific intent crime, in that the defendant must intend to permanently deprive the person of his interest in the property. Importantly, this intent generally must exist at the time of taking (save for the continuing trespass doctrine). In this case, the friend believed that the mechanic was being unlawfully deprived of his property, and his intent was to return the tools to their rightful owner. Thus, when he took the tools, he lacked the intent to permanently deprive another of his interest in the property. This would be true even if the garage owner factually had a lawful right to retain the tools, as the mistake of law would negate the friend's intent to permanently deprive another of his interest in the property. (A) is wrong, of course, because he did not have the consent of the owner of the tools he took. (C) is wrong because it is not a defense to a charge of larceny that the defendant did not intend to keep the item taken for his own use. (D) is wrong because it prematurely raises the continuing trespass doctrine. Under the continuing trespass doctrine, a defendant may be convicted of larceny when he initially takes property with a wrongful state of mind, but without the intent to permanently deprive the owner of the property, and then later forms the intent to permanently deprive the owner of the property. Here, the friend had not yet had the opportunity to form that later intent (even assuming the initial taking was with a wrongful state of mind). Thus, (D) does not address the correct issue.

A state's penal statutes contain the following: "Any person who commits a battery on a state official knowing that the victim is a state official shall be punished by imprisonment in the county jail for not more than one year or fined not more than $5,000, or both." While picking up her unemployment check at the state office building, a woman got into an argument with a man wearing a brown suit and angrily shoved him against the wall, causing him no injuries. The man she shoved was the director of the state department of unemployment, and the woman is prosecuted under the penal statute quoted. The woman admits that she shoved the victim intentionally but denies that she knew the man was a state official. What verdict should the jury return? A. Not guilty, if they find that the state should have warned the woman that the victim was an official, even though they do not believe her testimony. B. Not guilty, if they believe the woman's testimony. C. Guilty, even if they believe the woman, because this is a public welfare offense. D. Guilty, regardless of whether they believe the woman, because she admitted that she intentionally shoved the victim.

B The jury should return a verdict of not guilty if they believe the woman's testimony. The statute punishes a battery committed against a victim the defendant knows is a state official. Therefore, if the jury believes the woman's testimony that she did not know the man was a state official, they must find her not guilty of the charged violation. This is not a public welfare offense, but a particular form of battery. Thus, (C) is incorrect. (D) is incorrect because the statute requires knowledge of the status of the victim. (A) is wrong. There is no duty on the part of the state to warn the woman. If she knew, she will be guilty; if she did not know, she will not be guilty.

The police set up a sting operation targeting a resale shop that had a reputation for selling stolen goods. An undercover police officer approached the owner of the shop posing as a truck driver who was down on his luck and looking for a way to earn some extra money. The shop owner suggested that on his next load the officer should take a box of goods from the back of his truck, bring them to the shop owner to sell in exchange for cash, and then report the box lost to the trucking company. The next day the officer gave the shop owner an empty box in exchange for money. Immediately after the exchange, the shop owner was arrested. Which of the following crimes did the owner commit? A. Receipt of stolen property. B. Attempted receipt of stolen property. C. Conspiracy and receipt of stolen property. D. No crime.

B The owner committed attempted receipt of stolen property. One commits the crime of receipt of stolen property by receiving possession and control of "stolen" personal property, known to have been obtained in a manner constituting a criminal offense by another person with the intent to permanently deprive the owner of his interest in the property. The property must have "stolen" status at the time it is received by the defendant. Here, the defendant did not receive any actual stolen property, only an empty box. Thus both (A) and (C) are incorrect. (C) is also incorrect under the common law definition of conspiracy, which requires that there be an agreement between two or more conspirators. Unless the jurisdiction follows the unilateral approach (which does not require two or more culpable parties), the fact that the officer was only feigning agreement would preclude the shop owner's liability for conspiracy. (B) is the best answer. Criminal attempt is defined as an act that, although done with the intention of committing a crime, falls short of completing the crime. There must be a specific intent to commit the crime and an overt act in furtherance of that intent. The shop owner intended to receive stolen property and delivered a payment to the undercover officer in furtherance of that intent. While there was no actual receipt of stolen goods, had the circumstances been as the shop owner believed them to be he would have been guilty of receipt of stolen property. In most states and under the MPC, factual impossibility is not a defense to an attempt charge, so it is not a defense that there was no actual stolen property involved. Because (B) is correct, (D) is incorrect.

A robber attempted an armed robbery of a liquor store with an accomplice. A police officer arrived at the scene and shot and killed the robber's accomplice, who had been guarding the door with a gun and had not obeyed the officer's command to drop the weapon. Meanwhile, the store owner pulled out a gun and shot at the robber but missed. The robber then killed the store owner in self-defense before he could fire a second shot. In most jurisdictions, for whose death can the robber be found guilty under the felony murder doctrine? (A) The store owner and the accomplice. (B) The store owner only. (C) The accomplice only. (D) Neither the store owner nor the accomplice.

B The robber can be found guilty of felony murder of the store owner only. This choice represents an exception to the general rule that almost any death occurring in the course of a felony is felony murder. In the majority of jurisdictions, the robber would not be held guilty of felony murder for a justifiable killing of a co-felon by a police officer. The Redline view (the majority position) holds that the killing of a felon by a police officer or resisting victim cannot be the basis for felony murder. Thus, (A) and (C) are incorrect. (C) and (D) are incorrect because a person has no right of self-defense when he is the aggressor, and especially if he is engaged in a felony. The store owner, who was the victim of this felony, had the right to use at least the threat of force against the robber. Because the death occurred in the course of the felony, and the robber had no right of self-defense, he is guilty of felony murder, as well as deliberate premeditated murder.

A thief looking for targets in a hotel lobby one evening spotted the victim wearing what appeared to be expensive jewelry as she checked into the hotel. After finding out the victim's room number, the thief broke into a supply room and put on a bellhop's uniform. She then grabbed some flowers from a vase in the hall and knocked on the door to the victim's room, announcing the delivery of a bouquet of flowers. After the victim let her in, the thief scanned the room for the jewelry while putting the flowers in a vase. When she did not see the jewelry, she pulled out a knife and forced the victim to reveal the whereabouts of the jewelry, which turned out to be the hotel's safe. The thief made the victim call the front desk and ask that someone bring the jewelry to the room. The thief then locked the victim in the bathroom, changed out of the bellhop's uniform, and accepted the jewelry when it was brought to the room. She was apprehended a few days later trying to sell the jewelry. Under these facts, what are the most serious crimes the thief can be convicted of? (A) Burglary and larceny. (B) Burglary and robbery. (C) Larceny only. (D) Robbery only.

B The thief can be convicted of burglary and robbery. At common law, the elements of burglary are: (i) a breaking (ii) and entry (iii) of the dwelling (iv) of another (v) at nighttime, (vi) with the intent to commit a felony therein. Here, the thief has committed a constructive breaking because she gained entry by means of a fraud. The hotel room constitutes a dwelling for purposes of burglary, and the thief apparently had the intent to commit larceny when she entered the room. The thief has also committed robbery, which is defined as (i) a taking (ii) of personal property of another (iii) from the other's person or presence (iv) by force or intimidation, (v) with the intent to permanently deprive the other of the property. The "presence" element is satisfied if the victim is in the vicinity when the property is taken. The thief used the threat of force against the victim to obtain the property, and obtained it while the victim was locked in the bathroom of the hotel room, and therefore in the vicinity. Thus, she can be convicted of both burglary and robbery, making (C) and (D) incorrect. (A) and (C) are incorrect because the elements of larceny are contained within the more serious offense of robbery, which is basically an aggravated form of larceny

At a party, a woman and a man agreed to play a game they called "spin the barrel." The man took an unloaded revolver, placed one bullet in the barrel, and spun the barrel. The man then pointed the gun at the woman's head and pulled the trigger once. The gun did not fire. The woman then took the gun, pointed it at the man, spun the barrel, and pulled the trigger once. The gun fired, and the man fell over dead. A statute in the jurisdiction defines murder in the first degree as an intentional and premeditated killing or one occurring during the commission of a common-law felony, and murder in the second degree as all other murder at common law. Manslaughter is defined as a killing in the heat of passion upon an adequate legal provocation or a killing caused by gross negligence. The most serious crime for which the woman can properly be convicted is: A. Murder in the first degree, because the killing was intentional and premeditated and, in any event, occurred during commission of the felony of assault with a deadly weapon. B. Murder in the second degree, because the woman's act posed a great threat of serious bodily harm. C. Manslaughter, because the woman's act was grossly negligent and reckless. D. No crime, because the man and the woman voluntarily agreed to play a game and each assumed the risk of death.

B The woman is not guilty of first-degree murder under the provided statute because she lacked the requisite premeditated intent and did not commit one of the common-law felonies (i.e., burglary, arson, rape, robbery, or kidnapping) which support a felony murder conviction. Because the statute defines all murder that is not firstdegree murder as second-degree murder, the woman is guilty of second-degree murder, which makes (A) incorrect. (C) is incorrect because the question asks what the most serious crime could be, and manslaughter is a less serious crime than second-degree murder. Her act showed reckless indifference to human life, which supports a murder conviction. (D) is incorrect because consent is not a defense to murder

A man and woman agreed to burn down a neighbor's house in retribution for some wrong the neighbor allegedly committed against them. Both the man and woman were arrested shortly after they poured gasoline on the neighbor's front porch. The man revealed to the police that he participated in the plan to ensure that nothing bad would happen to the neighbor, and that he had made an anonymous telephone call to the police alerting them to the crime, which enabled the police to arrest him and the woman "in the act." The woman stated that she would not have participated if not for the man's encouragement. If the woman is charged with a conspiracy at common law to commit arson, how should she be found? A. Not guilty, because she was not predisposed to commit the crime but for the man's encouragement. B. Not guilty, because the man did not intend to commit arson. C. Guilty, because there was an agreement, and pouring gasoline on the front porch was sufficient for the overt act. D. Guilty, because arson is not a specific intent crime.

B The woman should be found not guilty of a conspiracy to commit arson. To be convicted of a conspiracy at common law, it must be shown that at least two persons agreed to achieve an unlawful objective. Having two or more persons is a necessary element of conspiracy under the traditional bilateral approach. Here, the facts indicate that the man did not intend to achieve the objective of the conspiracy—to burn the dwelling house of another. Thus, the woman cannot be guilty of conspiracy to commit arson. (C) is incorrect. The man feigned his agreement, making the answer factually inaccurate. (D) is also incorrect. Although it is true that arson is not a specific intent crime, conspiracy is a specific intent crime, in that the prosecution must show that the defendant intended to agree and intended to achieve the unlawful objective. Thus, the fact that the underlying crime is not a specific intent crime is irrelevant. (A) is incorrect. Even if the woman would not have committed the crime without the man's inducement, that is not a defense for the woman. A person cannot be entrapped by a private citizen.

The defendant was invited over to a friend's house one night for drinks. The friend told the defendant that he wanted to show him a pistol that he had stolen. After showing him the pistol, they decided to go out to a bar. As they got to the defendant's car, the defendant realized that he had left his coat in the house and told the friend that he was going back to get it. The defendant opened the front door and went to the room where his coat was. As the defendant picked up his coat, he saw the stolen pistol on the table. Realizing that he could get some quick cash by reselling it, he put it in his pocket and returned to his car. In a common law jurisdiction, of which crimes could the defendant be convicted? A. Both larceny and burglary. B. Burglary only. C. Larceny only. D. Neither larceny nor burglary.

C . In a common law jurisdiction, the defendant is guilty of larceny, but not burglary. At common law, larceny is the taking and carrying away of the tangible personal property of another by trespass with the intent to permanently (or for an unreasonable time) deprive the other of the property. Larceny is a trespass against possession, not ownership, and therefore, in most jurisdictions, a defendant can be convicted of larceny even when he takes property from someone who only has possession of that property because of some wrongful act. Thus, the defendant is guilty of larceny when he took the stolen pistol from his friend's house with the requisite 3 intent, and (B) and (D) are incorrect. However, the defendant would not be guilty of burglary. At common law, burglary is the breaking and entering of the dwelling of another at nighttime with the intent of committing a felony therein. In the instant case, the defendant did not enter the house with the intent to steal the pistol. Furthermore, a breaking requires a trespass. If the defendant has consent to enter, his use of force to gain entry is not a breaking. Thus, although opening a shut door may be enough to constitute a breaking, it would not be a breaking under these facts, because the defendant presumably had consent to enter to retrieve his coat. Therefore, (A) and (B) are incorrect.

The accused rented a room for two nights at a motor hotel. This room, like all others in the motor hotel, was equipped with a large color television set. The accused decided to steal the set, pawn it, and keep the proceeds. To conceal his identity as the thief, he contrived to make his room look as if it had been burglarized. However, he was traced through the pawnbroker and arrested. On these facts, the accused is guilty of: A. Embezzlement. B. False pretenses. C. Larceny. D. Larceny by trick.

C . The accused is guilty of larceny because, while having mere custody of the television set, he carried it away from the hotel intending to permanently deprive the hotel owner of his interest in the set. Larceny consists of the taking and carrying away of tangible personal property of another by trespass, with intent to permanently (or for an unreasonable time) deprive the person of his Interest in the property. Property must be taken from someone who has a possessory interest superior to that of the defendant. If the defendant has custody of the property, rather than possession, his misappropriation of the property is larceny. Possession involves a much greater scope of authority to deal with the property than does custody. Here, the accused only had the authority to use the television set for viewing purposes while he was staying at the hotel. Thus, the accused had only enough authority to deal with the set as to indicate that he had custody of it rather than possession. Consequently, the hotel owner had a possessory interest in the set superior to that of the accused. The accused took the set by trespass (without the consent of the owner) and carried it away with the intent to permanently deprive the owner of his interest in the set. Thus, the accused is guilty of larceny. (D) is incorrect because larceny by trick occurs when the victim consents to the defendant's taking possession of the property but such consent has been induced by a misrepresentation, Here, the hotel owner never consented to give the accused possession of the television set, through misrepresentation or otherwise. Instead, the accused simply took the set without the consent of the owner. Therefore, this is not larceny by trick. (B) is incorrect for a similar reason. The offense of false pretenses consists of obtaining title to the property of another by an intentional or knowing false statement of past or existing fact, with intent to defraud the other. The accused made no misrepresentations to the hotel owner, nor did the owner convey title to the television set to the accused. Thus, the accused is not guilty of false pretenses. (A) is incorrect because embezzlement requires the fraudulent conversion of property of another by a person in lawful possession of that property. The accused never had lawful possession of the television set. The taking of the set without the consent of the hotel owner was trespassory. Thus, the accused has not committed embezzlement.

A wealthy bicycle aficionado loaned one of his bicycles to a cyclist for use in a race. After the race, the cyclist failed to return the bicycle despite repeated requests by the aficionado. Several months later, while walking through the park, the aficionado happened upon the cyclist riding another bicycle that was the same make, model, and color as the aficionado's bicycle. Believing it to be his bicycle, the aficionado, standing only inches away from the bicycle, verbally threatened to grab the bicycle out from under the cyclist if he did not hand it over immediately. The cyclist, fearful of such an attack, surrendered the bicycle to the aficionado. Of the following, what is likely the most serious offense of which the aficionado could be properly convicted? (A) Robbery. (B) Larceny. (C) Assault. (D) No crime.

C Assault is (i) an attempt to commit a battery, or (ii) intentionally placing another in apprehension of imminent bodily harm. Battery is the (i) unlawful (ii) application of force (iii) to another person (iv) that causes bodily harm to that person or (v) constitutes an offensive touching. With regard to the "fear of harm" type of assault, although the cyclist was fearful of the aficionado carrying out his threat, the threat was not to inflict imminent bodily harm on the cyclist, but instead to damage the bicycle. By contrast, with regard to the attempted battery version of assault, the verbal threat made by the aficionado coupled with his proximity to the bicycle makes it likely that he took a substantial step towards committing a battery. Although the aficionado did not threaten to harm the cyclist, he did threaten an offensive touching of the bicycle which was attached to the cyclist's person. The cyclist's apprehension of this offensive touching occurring was likely reasonable, given the aficionado's verbal threat and proximity to the bicycle. Even though he made a reasonable and honest mistake of fact that the bicycle in question was his bicycle because both were the same make, model, and color, the aficionado has no right to recover his own bicycle by the use of force when he voluntarily relinquished possession of it to the cyclist initially. Consequently, although the aficionado did not commit the "fear of harm" type assault, he did commit the attempted battery type of assault. Answer choice B is incorrect. 8 Larceny is the (i) trespassory (ii) taking and carrying away (iii) of the personal property of another (iv) with the intent to permanently deprive that person of the property (i.e., intent to steal). A mistake of fact is a defense to a specific-intent crime, even if the mistake is unreasonable. Here, the aficionado had a reasonable and honest belief that the bicycle in question was his bicycle. Consequently, he did not have the specific intent to take the personal property of another and therefore did not commit larceny. Answer choice A is incorrect. Robbery is larceny from the person or presence of the victim by force or intimidation. Although the aficionado did gain possession of the bicycle through intimidation, because he did not commit larceny, he also cannot have committed robbery. Answer choice D is incorrect because, although the aficionado did not commit robbery or larceny, he did commit assault.

A woman and a man were living together in a house owned by the man when they broke up. The man told the woman to leave and changed all the locks in his house. The woman, wanting to take the man's television to punish him, asked her friend to help her, telling her friend that the woman owned the television. One night, the friend helped the woman pick the lock on a back door while the boyfriend was away. Once inside, the woman pointed out the television. The friend carried the television out to the woman's car. She drove to her new apartment where her friend brought the television inside. When the man discovered the television was gone, he broke into the woman's new apartment one night, and retrieved the television. The jurisdiction continues to follow the common law with regard to the crime of burglary. Given these facts, which, if any, of these individuals is guilty of burglary? (A) None of them. (B) The friend and the woman. (C) Only the woman. (D) Only the man.

C Burglary is the breaking and entering of the dwelling of another at nighttime with the specific intent to commit a felony therein (in this case, larceny). Persons taking back their own property or taking property in the honest but mistaken belief that the property belongs to someone who has authorized them to take it lack the intent to steal required for larceny. In this case, all of the elements of burglary are satisfied as to all three individuals, except for the specific intent requirement. The friend and the man lacked the requisite criminal intent for burglary. Only the woman had the requisite specific intent. The man's house was no longer her dwelling. Although she did not remove the television from the man's house, she entered the house with the specific intent to deprive the man of the television (i.e., to steal). Answer choice A is incorrect because the woman had the requisite criminal intent for burglary. Answer choice B is incorrect because the friend thought he was helping the woman retrieve her property, and therefore lacked the requisite intent for burglary. Answer choice D is incorrect because the man was retrieving his property, and therefore lacked the requisite intent for burglary

A man broke into and entered a home at night with the intent of bludgeoning the homeowner to death with a baseball bat. Upon seeing the man raise the baseball bat to strike him, the homeowner pled for his life. The man decided not to strike or kill the homeowner and instead took personal property worth several thousand dollars from the home. Of the following, which are the most serious crimes for which the man can be convicted? (A) Robbery and larceny only. (B) Burglary and larceny only. (C) Burglary and robbery only. (D) Burglary, robbery, and larceny.

C Robbery is larceny from the person or presence of the victim by force or intimidation. Larceny merges into robbery. Thus, if a defendant has committed both robbery and larceny, the defendant can only be convicted of robbery. Burglary is the (i) breaking and (ii) entering (iii) of the dwelling (iv) of another (v) at nighttime (vi) with the specific intent to commit a felony therein. A defendant who fails to commit the underlying felony is nevertheless guilty of burglary. Consequently, the most serious crimes for which the man can be convicted are robbery and burglary. Answer choice A is incorrect because a defendant who has committed both robbery and larceny can only be punished for one of these crimes. Answer choice B is incorrect. A defendant can be convicted of both burglary and the underlying felony. However, the call of the question asks for the most serious crimes for which the man can be convicted. Thus, even though the man committed larceny, the man also committed robbery (larceny by force or intimidation) which is a more serious crime than larceny. Consequently, the most serious crimes for which the man can be convicted are robbery and burglary. Answer choice D is incorrect because the man cannot be punished for both larceny and robbery as larceny merges into robbery

One afternoon, a woman was walking down the street when a man walked up behind her, grabbed her purse, and ran away. That evening, as the woman and her husband dined at a restaurant, she recognized an identical purse hanging from the back of a chair upon which another woman was sitting at a table a few feet away. She pointed the purse out to her husband before going up to the table to confront the couple sitting there. The couple claimed that the purse was certainly not stolen and refused to give it to her, so the woman violently yanked it from the chair, knocking it and the woman sitting on it over, and returned to the table with her husband. Robbery charges were subsequently brought against her, and during trial, evidence indicated the purse did actually belong to her. Is the woman likely to be successfully convicted of robbery? (A) Yes, because she stole the property at night. (B) Yes, because she took the purse by force. (C) No, because the purse had been stolen from her earlier in the day. (D) No, because she took the purse from the chair, not the other woman.

C Robbery is larceny from the person or presence of the victim by force or intimidation. The force used by the defendant must be more than the amount necessary to effectuate taking and carrying away the property. Here, the woman did use force to take the property from another, with the intent to permanently deprive her of that property, but because the property was hers to begin with, she did not take property that belonged to another. Answer choice A is incorrect because robbery need not occur at night. The answer alludes to the requirements for common law burglary. Answer choice B is incorrect because while the woman did use force to obtain the purse, that is only one element of robbery, not all of which are met here. Answer choice D is incorrect because, while the force need not be applied to the actual person if the other elements of robbery were met, those requirements were not met here

The accused was the owner of a furnished cottage, which she leased to a tenant for one year. While this lease was in effect, the accused found herself in immediate need of cash, and decided to burn down the house to collect the insurance on it. She waited until one evening when the tenant was away at the theater, and then used her own key to gain access to the house. To make it appear that the fire was caused accidentally by the tenant, she soaked one end of the mattress on the bed in the bedroom with gasoline and then left a lighted cigarette burning at the other end of the mattress. She planned that the cigarette would ignite the mattress and that when the fire smoldered to the area soaked in gasoline, the entire bed would burst into flames, and the resulting fire would destroy the house. However, the tenant returned home earlier than expected and discovered the fire just as the mattress burst into flames. The tenant immediately put it out with a fire extinguisher. A police investigation revealed the accused's activities. The accused is guilty of: (A) Burglary as to the house and arson as to the mattress. (B) Neither burglary nor arson because the accused owned the structure and its contents. (C) Burglary and attempted arson. (D) Attempted arson but not burglary because the accused entered with her own key.

C The accused is guilty of burglary because the right of occupancy belonged to the tenant. However, the fact that there was no burning of the structure means that the accused is guilty of attempted arson, rather than arson. Burglary is a breaking and entering of the dwelling of another at nighttime, with the intent of committing a felony therein. A breaking requires some use of force to gain entry, but minimal force is sufficient. In determining whether the dwelling is that of another, occupancy rather than ownership is material. Thus, an owner can commit burglary of her own structure if it is rented and used as a dwelling by someone else. Here, although the accused owned the cottage, the tenant had the right to occupy it pursuant to a lease. Thus, for purposes of the crime of burglary, the accused is deemed to have entered the dwelling of another. Although the accused used her own key to gain access to the cottage, this was still an unconsented use of force to effectuate entry, thereby constituting a breaking. This breaking and entering of the tenant's dwelling occurred in the evening. At the time of the entry, the accused intended to commit the felony of arson. Consequently, all the elements of burglary are in place, making the accused guilty of this crime. Arson consists of the malicious burning of the dwelling of another. There is a requirement of some damage to the fiber of the wood or other combustible material. As with burglary, ownership of the structure is not material for determining whether the dwelling is that of another; rather, the right to occupancy is material. The accused left a lighted cigarette on the mattress, intending to burn down the entire cottage. However, the tenant extinguished the fire before any damage was done to the structure of the cottage, even mere charring. Absent such damage, arson cannot have been committed. The accused did commit attempted arson. A criminal attempt is an act which, although done with the intention of committing a crime, falls short of completing the crime. The defendant must intend to perform an act and obtain a result that, if achieved, would constitute a crime. Also, the defendant must have committed an act beyond mere preparation for the offense. The accused intended to perform an act that would have culminated in the crime of arson. By soaking the mattress with gasoline and leaving a lighted cigarette on it, the accused committed an act that came dangerously close to successfully burning the cottage. This act in combination with the intent to commit arson means that the accused is guilty of attempted arson. (A) is incorrect because there can be no arson as to the mattress. Arson requires a burning of a dwelling. Because the 3 cottage was not burned, the accused is not guilty of arson. (B) is incorrect because the key element in determining whether a dwelling is that of another, for both arson and burglary, is the right of occupancy. Under the terms of his lease, the tenant had the right to occupy the cottage for one year. Therefore, the accused's ownership of the cottage will not be a defense to either arson or burglary. (D) is incorrect because, as explained above, the accused's use of a key to gain access to the cottage without the consent of the person who had the right of occupancy is deemed to be a use of force to gain entry, in the same way as if a person who did not own the cottage were to gain entry by means of a skeleton key.

The accused rented a room for two nights at a motor hotel. This room, like all others in the motor hotel, was equipped with a large color television set. The accused decided to steal the set, pawn it, and keep the proceeds. To conceal his identity as the thief, he contrived to make his room look as if it had been burglarized. However, he was traced through the pawnbroker and arrested. On these facts, the accused is guilty of: (A) Embezzlement. (B) False pretenses. (C) Larceny. (D) Larceny by trick.

C The accused is guilty of larceny because, while having mere custody of the television set, he carried it away from the hotel intending to permanently deprive the hotel owner of his interest in the set. Larceny consists of the taking and carrying away of tangible personal property of another by trespass, with intent to permanently (or for an unreasonable time) deprive the person of his Interest in the property. Property must be taken from someone who has a possessory interest superior to that of the defendant. If the defendant has custody of the property, rather than possession, his misappropriation of the property is larceny. Possession involves a much greater scope of authority to deal with the property than does custody. Here, the accused only had the authority to use the television set for viewing purposes while he was staying at the hotel. Thus, the accused had only enough authority to deal with the set as to indicate that he had custody of it rather than possession. Consequently, the hotel owner had a possessory interest in the set superior to that of the accused. The accused took the set by trespass (without the consent of the owner) and carried it away with the intent to permanently deprive the owner of his interest in the set. Thus, the accused is guilty of larceny. (D) is incorrect because larceny by trick occurs when the victim consents to the defendant's taking possession of the property but such consent has been induced by a misrepresentation, Here, the hotel owner never consented to give the accused possession of the television set, through misrepresentation or otherwise. Instead, the accused simply took the set without the consent of the owner. Therefore, this is not larceny by trick. (B) is incorrect for a similar reason. The offense of false pretenses consists of obtaining title to the property of another by an intentional or knowing false statement of past or existing fact, with intent to defraud the other. The accused made no misrepresentations to the hotel owner, nor did the owner convey title to the television set to the accused. Thus, the accused is not guilty of false pretenses. (A) is incorrect because embezzlement requires the fraudulent conversion of property of another by a person in lawful possession of that property. The accused never had lawful possession of the television set. The taking of the set without the consent of the hotel owner was trespassory. Thus, the accused has not committed embezzlement.

After a business lunch at a restaurant, a patron went to the restaurant's cloakroom to get his coat. He removed a coat he believed to be his own from the coat rack. The coat was of similar color and of the same brand as the patron's own coat, but it actually belonged to another customer. The patron walked back to his office and removed the coat. As he was hanging it up, he noticed that another person's name was written on the inside of the collar. He immediately left the office and sprinted back in the direction of the restaurant, intending to return the coat and pick up his own. As he crossed the street, the patron was struck by a car. Although his injuries were minor, the coat was destroyed. If the patron is acquitted of larceny, what is the best reason for his acquittal? (A) The coat was destroyed through no fault of his own. (B) The coat looked so much like the patron's coat that his mistake was reasonable. (C) The patron thought the coat was his when he took it. (D) When he realized the coat was not his, the patron tried to return it.

C The best reason for the patron's acquittal is that he thought the coat was his when he took it. Because the patron believed the coat to be his own, he lacked the intent to commit larceny. Larceny consists of a taking and carrying away of the tangible property of another by trespass, with the intent to permanently deprive the person of his interest in the property. The intent to deprive the owner of his property generally must exist at the time of the taking (except for the continuing trespass situation; see below). Here, the patron took and carried away a coat belonging to another. But when he took it, the patron believed that the coat was actually his own coat. Thus, the patron did not take the coat with the intent to permanently deprive the owner of his property. Absent this intent, the patron cannot be guilty of larceny. (A) is incorrect because although the patron did not actually destroy the coat himself, he could still be guilty of larceny. As mentioned, larceny requires the intent to permanently deprive the owner of the property. This intent includes intentionally dealing with property in such a way as to create a substantial risk of loss. Thus, even if the patron did not destroy the property himself, if he dealt with it in such a way as to risk its destruction, he could still be guilty of larceny, and so this is not the best reason for his acquittal. (B) is incorrect because the reasonableness of the mistake is not relevant. The test for whether a 14 defendant has the intent for larceny is a subjective one—what this defendant intended. Thus, even if the mistake were unreasonable, the patron would not be guilty of larceny if he believed the coat was his. (D) is incorrect because it attempts to negate the continuing trespass situation, which is not applicable here. Under the continuing trespass theory, if a defendant wrongfully (i.e., with a "bad" mental state) takes the personal property of another without the intent to permanently deprive the owner, but later, while still in possession of the property, decides to keep it, he is guilty of larceny. This theory does not apply here because the patron's taking of the coat was not wrongful. Thus, it is irrelevant that he was trying to return the coat; even if he had decided to keep it, he would not be guilty of larceny, although he would be guilty of some other crime. Therefore, (D) is not the best reason for the patron's acquittal because it raises irrelevant issues.

A shopper at a flea market noticed a vase and asked a nearby person how much it cost. The person replied, "One hundred dollars." The shopper paid him the money and took the vase. The person, who was not the owner of the vase but merely a bystander, absconded with the $100. What crime at common law has the bystander committed with respect to the $100? A. Embezzlement. B. Larceny by trick. C. Obtaining property by false pretenses. D. No crime.

C The bystander has obtained property by false pretenses. In the crime of false pretenses, the defendant obtains title to the property by means of a false representation of a material present or past fact that causes the victim to pass title to his property to the defendant, who knows his representations to be false and intends thereby to defraud the victim. Under the circumstances, by replying that the price of the vase was $100, the bystander implied that he was its owner or was authorized to sell it, which he was not. This false representation induced the victim to pass title to his money to the bystander, who knew the representation to be false. Therefore, he is guilty of obtaining money by false pretenses and (D) is incorrect. (A) is incorrect because the bystander took title, not mere possession. Therefore, the crime is false pretenses, not embezzlement. The physical transfer of that cash without any limitations on its use was a transfer of title to the money. Therefore, the crime is the obtaining of money by false pretenses. (B) is incorrect because larceny by trick occurs when the defendant obtains possession of another's property by lying or trickery. Here, the bystander obtained title to, rather than mere possession of, the money, and is therefore not guilty of larceny by trick.

A cashier at a bookstore who accepted a $50 bill as payment from a customer placed the bill underneath all of the $20 bills in the cash register and took it home with her at closing time. At the cashier's second job as a truck driver, she drove her tractor to a trucking terminal and attached a trailer filled with wine. At the first truck stop, the cashier entered the trailer, took a bottle of wine from a case, and drank it. In a common law jurisdiction, of which of the following crimes is the cashier guilty? (A) Larceny for the money and embezzlement for the wine. (B) Embezzlement for the money and larceny for the wine. (C) Larceny for both the money and the wine. (D) Embezzlement for both the money and the wine.

C The cashier is guilty of larceny in both cases. Larceny is the taking and carrying away of tangible personal property of another by trespass, with the intent to permanently deprive the other of his interest in the property. The cashier is guilty of larceny in the case of the money, even though she originally had possession of the fifty dollar bill when she first received it from the customer. If she had converted it at that time, she would have been guilty of embezzlement because the money never reached the possession of her employer. However, when she placed the bill in the cash register, the employer then obtained possession of the bill, and the cashier's rights over the money were reduced to custody. When she took the bill from the cash register at the end of the day, she committed a trespassory taking from her employer's possession and therefore committed larceny. (B) and (D) are therefore incorrect. The cashier is also guilty of larceny in the case of the wine. Here, as a bailee she clearly was in possession of the entire trailer. However, when she entered the trailer and took one wine bottle, she broke bulk, and possession of the wine bottle is deemed to revert back to the owner of the wine. Thus, when she removed that bottle from its place in the trailer with the intention of depriving its owner of the wine permanently, she committed larceny; hence, (A) and (D) are incorrect.

The defendant was fired from his sales job while calling on customers in another city. He failed to return the company car that he was using for his sales visits; instead, he sold the car to a "chop shop" for cash. As to the car, what crime has the defendant committed? A. Larceny. B. Larceny by trick. C. Embezzlement. D. Theft by false pretenses.

C The defendant has committed embezzlement because a court will probably find that he had "possession" of the car at the time he appropriated it. Embezzlement is the fraudulent conversion of the property of another by a person in lawful possession of it. In the instant case, the car belonged to the company for which the defendant worked, but the company probably gave the defendant lawful possession of it so that he could make sales calls. When he sold the car rather than return it, he wrongfully converted the car. This is embezzlement. (A) is incorrect. Larceny is the taking and carrying away of the property of another by trespass with the intent to permanently deprive the person of the property. The intent to permanently deprive must be concurrent with the taking and carrying away. In the instant case, the defendant did not have the intent to permanently deprive the company of the car when he was given the car. Thus, the intent element is missing. (B) is also incorrect. Larceny by trick is a specialized form of larceny. For larceny by trick, the defendant must acquire possession of the property by some misrepresentation concerning a present or past fact. The defendant did not come into possession of the car by misrepresentation, making (B) incorrect. For similar reasons, (D) is also incorrect. The crime of false pretenses is the obtaining of title to the property of another by an intentional (or knowing) false statement of past or existing fact with the intent to defraud. As with larceny by trick, the defendant here came into possession of the vehicle without use of a misrepresentation. Furthermore, he also never obtained title to the car. Thus, (D) is an incorrect answer.

A defendant lost some money while playing poker with several people at his friend's house. When the defendant accused his friend of cheating, his friend asked him to leave. The defendant became abusive and refused to leave, so his friend and a couple of other players forced him to go. Angry and determined to get back his money, the defendant went to his home and picked up his pistol. He headed back to his friend's house, intending to shoot his friend if he did not give back the money. However, due to the altercation at the house, his friend had called the police. Just as the defendant was about to step onto his friend's property, the police pulled up and stopped him. They frisked him, found the pistol in his pocket, and arrested him. A state statute prohibits entry onto the property of another with the intent to commit violence thereon. If charged with attempt under this statute, most likely the defendant will be found: A. Not guilty, because this is an "attempt" statute, and there cannot be an attempt of an attempt. B. Not guilty, because it would be an attempt to convict a person for a guilty mind. C. Guilty, because the defendant was trying to enter the property and he had the necessary state of mind. D. Guilty, because the statute was designed to protect the public from violence, and the defendant is dangerous.

C The defendant will likely be found guilty. An attempt requires both a specific intent to commit the crime and an overt act in furtherance of that intent. Given that the defendant intended to enter his friend's property and was apprehended just before doing so, both requirements for attempt can be established. (A) is incorrect because the defendant is not being charged with the attempt to commit violence, but with the attempt to enter onto the property with the intent to commit violence. (B) is also wrong because the statute prohibits not only the state of mind, but also an act (entering onto another's property). (D) is a poor answer because the defendant cannot be convicted unless it is proved that he committed the offense with which he has been charged, and this answer does not require that the elements of the crime be proved.

A father was angry at his son's coach because the coach would never let the son into a game. In order to exact revenge, the father decided to plant an incendiary device on the coach's front porch. The father believed the device would start a fire that would destroy the coach's home and perhaps injure him as well. However, the father made a mistake while assembling the incendiary device, and it was impossible for the device to do any harm. When the device went off, it did nothing more than produce a foul odor. If the father is charged with attempted murder and attempted arson in a common law jurisdiction, which of the following decisions is most likely to be reached by the court? A. The father is guilty of attempted murder and attempted arson. B. The father is guilty of attempted murder, but he is not guilty of attempted arson. C. The father is not guilty of attempted murder, but he is guilty of attempted arson. D. The father is not guilty of attempted murder or attempted arson.

C The father lacked the specific intent to kill that is required for attempted murder. However, the circumstances surrounding the "incendiary device" constitute factual impossibility and will not afford the father a defense to attempted arson. Criminal attempt is an act that, although done with the intention of committing a crime, falls short of completing that crime. To be guilty of attempt, the defendant must have the intent to perform an act and obtain a result that, if achieved, would constitute a crime. Regardless of the intent that would suffice for the completed offense, attempt always requires a specific intent to commit the target offense. Also, the defendant must have committed an act beyond mere preparation for the offense. Here, to be guilty of attempted murder, the father must have had the specific intent to kill his son's coach, even though the intent to inflict great bodily injury would be sufficient mens rea for murder. However, the facts indicate that the father intended at most only to injure the coach rather than kill him. Thus, the father cannot be guilty of attempted murder. However, the father did intend to burn the coach's home; therefore, he had the specific intent to commit arson by means of placing an incendiary device on the coach's porch, and his placing the device was an act beyond mere preparation for this crime. Although the device could not have actually burned the coach's house, it is no defense to attempt that it would have been impossible for the defendant to complete his plan. This is factual impossibility and is not a defense. Thus, the father is guilty of attempted arson. 3 (A) and (B) are incorrect because the father did not have the specific intent to kill. (D) is incorrect because the father is guilty of attempted arson, as explained above.

Two people agreed to steal a valuable painting that they knew was hanging in the victim's home. One would wait in the car with the engine running to ensure a quick getaway, while the other would break into the victim's home and steal the painting. The burglar broke into the home and reached the victim's library, where the painting was hanging. On the desk he noticed a large vial that appeared to contain cocaine. Thinking he could sell the cocaine and split the proceeds with the getaway driver, the burglar grabbed the vial and stuffed it in his pocket. He then took the painting off the wall and hurried back to the waiting car. The police arrived at that moment and apprehended the pair. A search incident to arrest turned up the vial of cocaine in the burglar's possession. The getaway driver is charged with being an accomplice to the unlawful possession of cocaine with intent to distribute. Will the driver likely be found guilty? A. Yes, because she is liable for all crimes resulting from the conspiracy. B. Yes, because the conspiracy was to steal items for resale. C. No, because the conspiracy did not involve the possession or sale of cocaine. D. No, because the burglar was the person who possessed the cocaine.

C The getaway driver should be found not guilty. At common law, once a conspiracy has been entered into, each conspirator, by virtue of her participation in the conspiracy, may be charged with "aiding and abetting" the commission of crimes by her co-conspirators and therefore may be liable for those crimes as an accomplice. Even if the conspirator did not have the sufficient mental state for accomplice liability, a separate doctrine provides that each conspirator may be liable for the reasonably foreseeable crimes of all other co-conspirators that were committed in furtherance of the conspiracy. However, in the instant case, the getaway driver did not know of the 2 theft of the cocaine, and, as a result, would not have the requisite mental state as an accomplice. The conspiracy doctrine discussed above also would not apply, as the crime of possession of cocaine with the intent to deliver would neither be in furtherance of the burglary nor a reasonably foreseeable result of the burglary. (A) is incorrect because it is too broad a statement. A conspirator is not criminally liable for all crimes resulting from the conspiracy. As discussed above, to be charged with a crime that grew out of the conspiracy, the additional crime must be both foreseeable and committed in furtherance of the conspiracy. (B) is also incorrect. Although a bit of a close call, the crime of possessing cocaine with the intent to distribute is a separate act from burglary, and a crime involving drug possession is not a crime ordinarily arising out of this type of burglary. It is this lack of foreseeability that makes (B) an incorrect choice. Finally, (D) is incorrect. The criminal liability of a conspirator for acts of a co-conspirator arises under the theory of accomplice liability. An accomplice need not have performed the criminal act himself to be held criminally liable for the criminal act of the principal. Thus, the fact that the actual burglar meets the definition of a principal is irrelevant.

A homeowner decided to destroy his home by fire to collect the insurance money. To work up his courage, he had several drinks at a local bar. When he returned to his block that night, he was so intoxicated that he mistakenly believed that his neighbor's house, which was 20 feet to the right of his house and looked very similar, was his own house. He started a fire under the back porch and went off a short distance to watch it burn. Suddenly he realized that he had the wrong house. He ran back and grabbed a garden hose and was able to put out the fire with just some slight charring of the porch. If the homeowner is charged with arson in a jurisdiction retaining the common law rules, what is the likely verdict? (A) Not guilty, because he did not have the requisite intent to burn the dwelling of another. (B) Not guilty, because he realized his mistake before any burning of the dwelling occurred. (C) Guilty, because he acted with malice. (D) Guilty, because his intent to commit arson of his own house is transferred to his neighbor's house.

C The homeowner should be found guilty. Common law arson consists of the malicious burning of the dwelling of another. At common law, the state of mind required—malice—is satisfied not only by intentionally burning the dwelling of another but also by acting with reckless disregard of an obvious risk that the structure would burn. While many courts ordinarily require that the defendant be subjectively aware of the risk, they will find malice when the failure to be aware of the risk is due to voluntary intoxication. Even had the homeowner done what he intended, he would have put his neighbor's house in jeopardy of burning. The fact that his intoxication caused him to fail to recognize the risk would not be a defense. Nor could he raise a mistake of fact defense because mistake of fact must be reasonable to negate the existence of malice, and here the facts state that his mistake was caused by his intoxication. (A) is incorrect because the malice required for common law arson may be satisfied by something less than the intent to burn down the dwelling of another, and here malice is established. (B) is incorrect because he caused a burning of the back porch, which is part of the dwelling, with the requisite malice; his conduct once he realized his mistake is irrelevant to his guilt. (D) is incorrect because his intent to burn down his own house does not constitute an intent to commit arson, which at common law is the burning of the house of another. His intent to burn down his own house also constituted malice for purposes of the burning of his neighbor's house, but not because of the doctrine of transferred intent.

In an effort to curb underage drinking, the police staked out a liquor store near a college campus that was suspected of selling liquor to minors in violation of state law. They recorded a liquor store clerk being told by a minor that he was an underage student at the local university and that he would pay triple the marked price for a case of beer. The clerk readily agreed to the deal and took the money, but both parties were arrested before they could complete the transaction. In a jurisdiction following the unilateral approach to conspiracy, which of the following statements is correct? A. The minor can be convicted of solicitation and the liquor store clerk can be convicted of conspiracy to violate the statute. B. Both the minor and the liquor store clerk can be convicted of conspiracy to violate the statute. C. The minor cannot be convicted of either solicitation or conspiracy, but the liquor store clerk can be convicted of conspiracy to violate the statute. D. The minor cannot be convicted of either solicitation or conspiracy, and the liquor store clerk cannot be convicted of conspiracy.

C The liquor store clerk can be convicted of conspiracy in a unilateral jurisdiction even though the minor can be convicted of neither solicitation nor conspiracy. Under the unilateral (Model Penal Code) approach, the crime of conspiracy requires only one party to have genuine criminal intent, and may be shown by proof that the defendant agreed with another to commit a crime (even if that other person does not share the commitment). Thus, the fact that no other party to the conspiracy could be found guilty does not prevent the defendant from being convicted of conspiracy. Here, the fact that the minor is a member of the class that the statute was designed to protect prevents him from being found guilty of conspiracy; (B) is therefore incorrect. However, unlike under the common law bilateral approach, under the unilateral approach this fact has no bearing on the liquor store clerk's liability for 4 conspiracy. Thus, (C) is correct and (D) is incorrect. (A) is incorrect because the crime of solicitation is treated the same as conspiracy. If the solicitor could not be guilty of the completed crime because of legislative intent to exempt him, he cannot be found guilty of solicitation of the crime. Because the minor is a member of the class intended to be protected by the statute, he cannot be found guilty of soliciting the liquor store clerk to commit it.

A motorist drove home from work late one night, and fell asleep behind the wheel of his car. His car drifted across the middle of the road and struck another car. The other driver was killed instantly in the collision. Angered by the noise of the collision, a homeowner fired a gun out the window of his house at the car. The bullet struck and killed a bystander. Both the motorist and the homeowner were arrested and charged with common law murder. Which of the defendants likely would be found guilty? A. Both the motorist and the homeowner. B. The motorist. C. Neither the motorist nor the homeowner. D. The homeowner.

D The homeowner likely would be found guilty. At common law, murder was the unlawful killing of a human being with malice aforethought. Malice aforethought could be established with any one of the following states of mind: intent to kill; intent to cause serious bodily harm; the depraved heart killing (a reckless indifference to an unjustifiably high risk to human life); or the commission of a felony. The homeowner would be guilty of murder. Firing a gun out of his window at a car would demonstrate a reckless indifference to a high risk to human life. Thus, (B) and (C) are incorrect. It is unlikely that the motorist would be guilty of murder. While his action might be classified as negligent or even reckless, it would not represent a depraved heart (reckless indifference to life) state of mind. Thus, (A) and (B) are incorrect.

Two affluent women were shoe shopping together one afternoon in a high-end shoe store. After trying on numerous pairs of expensive shoes, the older woman settled on a pair of shoes and gave the saleswoman her credit card. The younger woman was still debating over a pair of equally expensive high heels. As they were leaving the store, the older woman noticed that the younger woman was still wearing the high heels that she had not purchased, and she had subtly pulled her bell-bottom pants over the shoes so that they were hidden from the saleswoman's view. Upon discovering that the shoes were missing, the saleswoman called the police, and they were able to locate the older woman through her credit card information. The police were never able to locate the younger woman. Should the older woman be convicted of larceny as an accomplice? A. Yes, because the woman facilitated the crime by failing to make any effort to stop the younger woman. B. Yes, because the older woman saw the younger woman hiding the shoes under her bell-bottom pants. C. No, because the older woman had no intent to facilitate the younger woman's theft of the high heels. D. No, because the younger woman was never prosecuted as a principal because the police could not find her.

C The older woman did not have the mental state necessary for accomplice liability. Under the majority and MPC rule, an accomplice is a person who, with the purpose of promoting or facilitating the commission of the offense, aids or abets a principal prior to or during the commission of the crime. Mere knowledge that another person intends to commit a crime and mere presence on the scene are not enough to make a person an accomplice. Here, the older woman had knowledge of the crime, but she did not intend for the crime to occur, nor did she aid or abet the younger woman when she stole the shoes. Answer choices A and B are incorrect because as stated above, the older woman did not have prior knowledge of the crime, nor did she intend to facilitate the crime. Answer choice D is incorrect because by modern statute in most jurisdictions, an accomplice may be convicted of a crime even if the principal is not tried, is not convicted, has been given immunity from prosecution, or is acquitted..

A college student lived in a very exclusive section of a suburb and had grown into a willful, undisciplined person. One of the student's neighbors purchased one of the few gold-plated sports cars ever built and parked it in his attached multi-car garage. The student, whose aptitude for electronics was as thoroughly developed as unlimited time and money made possible, had manufactured a radio device that could be programmed to activate any automatic garage door opener, and the student used the device to open the neighbor's garage one night. Using other specialized tools, the student cut a key to the car and drove it away, intending to cruise by a few of his friends' favorite nighttime haunts in the opulently exotic car and then return it to its owner with no one the wiser. After driving the car to a local bar and having several drinks, the student decided to take his friends for a ride to the nearby seacoast. When the party arrived at the bluff, above the beach, the student impulsively decided to push the car over the cliff, making a spectacular golden wreck of the sports car. The jurisdiction's penal statutes simply codified the common law without alteration of any kind. The student is guilty of: (A) No crime. (B) Burglary. (C) Larceny. (D) Both burglary and larceny.

C The student is guilty of larceny only. Larceny is the taking and carrying away of the tangible personal property of another by trespass, with the intent to permanently deprive the person of his interest in the property. Under the continuing trespass doctrine, if a defendant takes property with a wrongful state of mind but without the intent to steal, and later, while still in possession of it, forms the intent to steal it, the trespass involved in the initial wrongful taking is regarded as "continuing" and the defendant is guilty of larceny. Here, the student's initial wrongful taking continued to the time the student intentionally destroyed the car, thereby permanently depriving the owner of possession. Therefore, the student committed larceny and (A) is incorrect. Burglary requires the breaking and entering of the dwelling of another at nighttime with the intent to commit a felony therein. The student is not guilty of burglary because, when he entered the attached garage, he did not intend to commit a felony therein; he merely intended to borrow the car. Thus, (B) and (D) are incorrect.

A man approached a woman on a street, pulled out a gun, and said: "See that store over there? If you don't go over and set fire to it right now, I'm going to shoot you." The woman walked over to the store, with the man still pointing the gun at her, went inside to the store's office, poured gasoline on all the office furniture, and set fire to the office. Shortly thereafter, the owner of the store rushed in trying to save her important papers. In doing so, the store owner was overcome by smoke and died in the fire. The woman is charged with the criminal homicide of the store owner. The most likely result is that the woman will be found to be: (A) Guilty of murder. (B) Guilty of manslaughter. (C) Not guilty of murder, because duress is a defense to arson. (D) Not guilty of murder, because the woman never intended to kill anyone.

C The woman is not guilty of arson (since she acted under duress) and therefore she is not guilty of the store owner's homicide. A person is not guilty of an offense, other than homicide, if he performs the otherwise criminal act under the threat of imminent infliction of death or great bodily harm and his perception of the threat is reasonable. Here, the woman set fire to the store because the man ordered her to do so or he would shoot her. Since the man had a gun, the woman's perception of the man's threat of imminent death or great bodily harm was reasonable. Therefore, the woman acted under duress and is not guilty of arson. If the woman is not guilty of arson, then she cannot be guilty of murder. Murder requires a mental state of malice aforethought (i.e., an intent to kill or inflict great bodily harm, an awareness of an unjustifiably high risk to human life, or the intent to commit a felony). Here, the state's only basis for showing malice aforethought would be to use the felony murder doctrine. Under that doctrine, a killing, even an accidental one, that occurs during the course of a felony is murder. The only felony here is arson (which includes the burning of buildings other than dwellings in most jurisdictions), and since the woman is excused from criminal liability for that crime due to duress, she cannot be convicted of murder under the felony murder doctrine because she did not commit the underlying felony. Thus, (C) is correct and (A) is incorrect. (B) is incorrect because the woman did not commit either voluntary or involuntary manslaughter. Voluntary manslaughter is an intentional killing distinguishable from murder by the existence of adequate provocation. As explained above, the woman did not have the requisite intent for murder (malice aforethought) and thus she cannot be guilty of voluntary manslaughter. Nor is the woman guilty of involuntary manslaughter. Involuntary manslaughter consists of either criminal negligence or "unlawful act" manslaughter. Since the woman acted under duress, she was not criminally negligent nor was the burning a "malum in se" (inherently wrongful) act necessary for "unlawful act" manslaughter. Therefore, there is no basis to find the woman guilty of manslaughter. (D) is incorrect because one can be convicted of murder without intending to kill (e.g., in the case of a felony murder). Thus, 6 if the woman were guilty of arson, she could be found guilty of murder despite the fact that she did not intend to kill anyone.

A con artist asked his friend to introduce him to the town's banker so that he could apply for a loan to set up a hardware business. The friend, also a friend of the banker, arranged a meeting and later gave the con artist a glowing recommendation based on their long and deep friendship. When the banker approved a $25,000 loan, the friend was present at the signing of the loan papers and co-signed on the con artist's behalf. Unbeknownst to the friend, the con artist intentionally misrepresented his intentions as to the proceeds of the loan and his financial status, forging some documents used to verify his solvency. The con artist has been tried and convicted of obtaining money by false pretenses (a felony) and sentenced to state prison. If the friend is charged as an accessory to obtaining money by false pretenses, he should be found: A. Guilty, because he was present when the crime was committed and was thus a principal in the second degree. B. Guilty, because he encouraged and aided the con artist, and his ignorance of the con artist's insolvency is no defense to the charged crime. C. Not guilty, because he lacked the requisite mental state to be an aider and abettor. D. Not guilty, because his encouragement and aid was not the legal cause of the offense.

C To be liable as an aider and abettor (i.e., an accomplice), the defendant must encourage or aid in commission of the underlying crime with the intent to assist the principal and the intent that the principal commit the crime. Because the friend had no knowledge of the crime, he could not be an aider and abettor. (A) is wrong because mere presence at the scene of the crime is not enough to make the friend a principal in the second degree. (B) is wrong because, as stated, there must be intent or knowledge to be an aider and abettor. (D) is wrong because if the requisite intent is present, the defendant can be liable even though he did not "cause" the crime.

Immediately after she arrived home from work, a woman found her husband engaged in sex with a female who worked in the husband's office. Enraged, the woman retrieved a handgun from her dresser drawer. She fired the gun, intending to shoot her husband's co-worker. Her shot missed the co-worker, and instead killed her husband. The woman was charged with common-law murder of her husband. Based on the foregoing facts, should she be convicted? A. Yes, because the woman acted with reckless indifference with regards to her husband's life. B. Yes, because of the doctrine of transferred intent. C. No, because the act was provoked. D. No, because the woman's extreme temporary distress completely excuses her actions.

C Voluntary manslaughter is murder committed in "the heat of passion," that is, in response to adequate provocation. The woman's enraged mental state mitigates the crime from murder to voluntary manslaughter. (A) is incorrect because, while firing a handgun in a bedroom in the direction of the victim could arguably constitute the intent necessary for murder, the provocation would reduce the crime to voluntary manslaughter. (B) is incorrect because, while the doctrine of transferred intent could supply the necessary intent for the killing of the husband to constitute murder, the adequate provocation would reduce the crime to voluntary manslaughter. (D) is incorrect because a defendant's action in the "heat of passion" does not serve as a complete defense to any crime related to the killing. The woman could be convicted of voluntary manslaughter.

A bartender promised to pay her friend $100 if he would take her television and stereo from her house, so that she could report the items as being stolen to collect a settlement from the insurance company. Although the friend had visited the bartender's house on two previous occasions, he mistakenly broke into and entered her neighbor's house and took the neighbor's television and stereo. When he returned to the bar where the bartender was waiting for him, both were arrested by the police. If the bartender and her friend are tried for conspiracy, the court will find them: A. Not guilty, because the friend failed to take the bartender's property. B. Not guilty, because the friend, being in the wrong house, could not take the bartender's property. C. Guilty, because they actually took the neighbor's property. D. Guilty, because they intentionally agreed to defraud the insurance company

D A conspiracy requires an agreement between two or more people to accomplish an unlawful act or objective. The parties must intend to enter into the agreement and intend to achieve the objective of the agreement. The bartender and her friend agreed to defraud the insurance company and had the requisite intent. They also committed an overt act in furtherance of that intent. Thus, they can be convicted of conspiracy. (A) is wrong because the crime is complete upon agreement with the requisite intent; the objective does not have to be accomplished. (B) is wrong because impossibility is not a defense to conspiracy. (C) is wrong. Taking the neighbor's property could make them guilty of some other crime, but it is not necessary to convict them of conspiracy

To test his new, high-powered cross-bow, the defendant went out into the woods behind his home, which bordered the backyard of a house where the defendant knew several children lived and played. The defendant posted a target on a tree and began firing arrows at the target. As he was testing his cross-bow, the defendant heard the children playing nearby, but took no action to warn them as to the danger. Even though the children began to run directly behind the target the defendant continued testing his cross-bow. The defendant accidentally misaimed and shot an arrow to the side of the tree with the target, striking and killing one of the children. The defendant was convicted of murder for the death of the child. On appeal, the defendant argued that the evidence at trial was insufficient as a matter of law to support a conviction of murder What action should the appellate court take with regard to this appeal? A. Reverse the conviction and remand for a new trial, because the evidence was not sufficient for murder but will support a conviction of involuntary manslaughter. B. Reverse the conviction and remand for a new trial, because the evidence is not sufficient for murder but will support a conviction of voluntary manslaughter. C. Reverse the conviction and order the case dismissed, because the evidence showed no intent to kill by the defendant. D. Affirm the conviction, as the evidence was sufficient to support a conviction of murder.

D Conduct that manifests a reckless indifference to an unjustifiably high risk to human life, unaccompanied by the intent to kill, and that actually results in death, is known as "depraved heart murder." Here, the jury could reasonably have found on the evidence at trial that testing the cross-bow in an area where children were known to play constituted conduct that manifested a reckless indifference to an unjustifiably high risk to human life. (A) is incorrect. The jury could reasonably have concluded that the defendant's actions rose above mere criminal negligence, required for involuntary manslaughter, to a "depraved-heart" standard, which can form the basis for a finding of murder. (B) is incorrect. Voluntary manslaughter can only be found when the killing is intentional. Here, there is no indication of intent to kill. (C) is incorrect. As noted above, "depraved heart murder" does not require intent to kill, just conduct that manifests a reckless indifference to an unjustifiably high risk to human life.

A bank teller confessed to his colleague that he was concerned about being fired. The colleague suggested that he had similar concerns himself and had been thinking of how he might acquire extra cash in case he lost his job. The teller suggested a plan to take several hundred dollars from their supervisor's cash drawer, because the supervisor often forgot to lock the drawer when he went to lunch. After the teller took the cash, he would hand it off to his colleague during his lunch break, and the colleague would bring the money to his car to avoid having money on his person at work. The next day, the teller noticed that his supervisor had neglected to lock his cash drawer at lunch, and he carried out the plan. The colleague waited outside the bank, and then took the cash from the teller on his lunch break. He then hid some of the money in a separate stash he intended to apportion to himself only, without the teller's knowledge, but he otherwise followed the plan exactly as he and the bank teller had discussed. In a case against the colleague, which of the following charges would most likely be successfully prosecuted? (A) Embezzlement and larceny. (B) Embezzlement only. (C) One charge of larceny only. (D) Two charges of larceny only.

D Larceny is the trespassory taking and carrying away of the personal property of another with the intent to permanently deprive that person of the property. An accomplice is a person who, with intent that the crime be committed, aids or abets a principal prior to or during the commission of the crime and is present at the commission of the offense. The accomplice's assistance to the principal may be verbal encouragement, financial assistance, or physical assistance, provided that the accomplice has the requisite intent to encourage or assist in the commission of the crime. An accomplice is responsible for the crime to the same extent as the principal. In this case, although the colleague did not actually take the money from the supervisor, he aided and abetted the teller in larceny by transferring the money to the car, and thus would be guilty to the same extent as the teller. The colleague also committed larceny against the teller, as stolen property taken from a thief can constitute larceny. That is, a second thief is guilty of larceny when he steals the property from the first thief, unless the second thief had superior possessory interest in the property. Therefore, answer choice C is incorrect. Answer choice A is incorrect because the teller would not be guilty of embezzlement, which is the fraudulent conversion of the property of another by a person who is in lawful possession of the property. Lower-level employees can generally not be guilty of embezzlement, as such an employee typically has custody, not possession, of the employer's property. In this case, the teller took the money from his supervisor, who was the one who had possession of the property. Accordingly, he would not be guilty of embezzlement, and thus his colleague would not either. Answer choice B is incorrect because it inaccurately states that the colleague would be guilty of embezzlement and fails to address that he aided in the commission of larceny.

While shopping in a department store, a man saw an expensive wallet on display, and slipped it into his girlfriend's purse without her knowledge as a practical joke to embarrass her. He intended for her to find it as she reached into her purse to pay for other items at the cashier's station and turn it over to the cashier. The couple shopped for the next hour, and the man forgot about the wallet. Before they could pay for their purchases, the couple got into an argument, abandoned their intended purchases, and headed toward an exit. When they attempted to leave the store, they were detained by a store security officer who found the wallet in the purse. Both parties were charged with larceny. Of the following, which would provide the man with his best defense to larceny? (A) The woman, and not the man, carried the wallet away from the display. (B) There was not a trespassory taking of the wallet since the man was on the store premises as a customer. (C) The wallet was never taken from the store. (D) The man lacked the necessary intent to steal the wallet.

D Larceny requires the intent to permanently deprive another person with superior rights to the property of that property. In this case, the man did not intend to permanently deprive the department store of the wallet. Rather, he intended for the girlfriend to return the wallet when she found it at the cashier's station. Therefore, the man did not possess the specific intent required for the crime of larceny. Answer choice A is incorrect because larceny can occur if property is taken and carried away by an agent, even if the agent is unaware that she is doing so. Answer choice B is incorrect because a trespassory taking requires only that the taking occur without the owner's permission, and not that the defendant be a trespasser when the taking occurs. Answer choice C is incorrect because, although the couple was detained before leaving the store and the wallet never left the premises, larceny requires only that the property be carried a short distance, provided the necessary intent exists at that time. Here, if the necessary intent had been present, a larceny would have been complete as soon as the wallet was carried away from the display case.

A man was asleep at table in a food court at a mall. While he slept, his hand rested on his wallet. A passerby, who was in the mall to shop, seized the opportunity to steal the man's wallet, slightly lifted the man's hand, and took the wallet. The man, waking up as the passerby turned to leave, noticed that the passerby had the wallet. The passerby ran and the man gave chase. The passerby threw the wallet down in front of an industrial floor cleaning machine that was being used to clean the floor. The machine destroyed the wallet. The passerby was arrested and charged with robbery. Of the following, which would constitute the passerby's best defense? (A) The passerby did not keep the wallet permanently. (B) The passerby did not act with premeditation. (C) The passerby did not harm the man. (D) The passerby did not use force or intimidation to gain possession of the wallet.

D Robbery is larceny from the person or presence of the victim by force or intimidation. Larceny is the (i) trespassory (ii) taking and carrying away (iii) of the personal property of another (iv) with the intent to permanently deprive that person of the property (i.e., intent to steal). Here, because the passerby took the man's wallet from the man while he was asleep, the passerby did not gain possession of the wallet through intimidation. In addition, although even the use of slight force to gain possession of another's property is generally sufficient, the force used by the defendant must be more than the amount necessary to effectuate taking and carrying away the property. Here, the passerby only used the force necessary to take and carry away the wallet (i.e., lifting the man's hand). Answer choice A is incorrect. Although a defendant must have the intent to permanently deprive the victim of the property, there is no requirement that the defendant retain the property permanently. The passerby's discarding of the wallet moments after taking it does not negate the requisite intent. Answer choice B is incorrect. Although a defendant must usually have the necessary intent at the time of the taking and carrying away of the property, there is no requirement that the defendant have such an intent only after reflecting on the idea of stealing the property or planning to do so. Answer choice C is incorrect. Although one of the ways by which a larceny becomes a robbery is through the defendant's use of force, there is no requirement that the defendant harm the victim in exercising such force.

A runner was in the midst of his morning run when the defendant jumped out from behind a bush and tripped the runner, intending to steal the runner's wallet. The runner was not carrying a wallet, however. Frustrated, the defendant tore off the runner's hat. The runner tried to grab the hat back from the defendant, but the defendant pulled it away and ran off. The struggle caused the defendant to lose his balance, and he dropped the hat a few feet from the runner. The runner then picked up his hat and completed his run. What is the most serious crime, listed in order of increasing seriousness, for which the defendant may be convicted? (A) Attempted larceny. (B) Larceny. (C) Battery. (D) Robbery.

D Robbery is larceny from the person or presence of the victim by force or intimidation. The force need not be great, but must be more than the amount necessary to effectuate taking and carrying away the property. The elements of larceny were met as soon as the defendant carried away the hat. Moreover, the force element was satisfied when the defendant tripped the runner and then struggled with the runner. Answer choices B and C are incorrect because battery and larceny are less serious than robbery. Although the elements of each of these crimes are satisfied, they would merge into the crime of robbery. Answer choice A is incorrect because larceny merely requires a taking and carrying away of the personal property of another with the intent to steal. These elements were satisfied as soon as the defendant took away the hat, so the defendant actually completed the crime of larceny.

A foreman worked as a third-shift supervisor at a power plant. One of his duties was to ensure that all timekeeping records accurately reflected the time his crew actually worked. All workers, including the company's accountant, were then paid for whatever hours the timecards reflected. The accountant was also required to assist in submitting budgets for payroll. Needing to leave work early for a second job that he obtained, the accountant had one of his trusted co-workers punch him out at the regular time every day of the week. At the end of the week, the accountant signed the timecard with the extra hours added. The accountant continued to do this for three weeks before one of his co-workers informed a human resources manager, who determined that the accountant's conduct resulted in an overpayment of approximately $1,000 to the accountant. What crime has the accountant committed? (A) Forgery. (B) Embezzlement. (C) Larceny by trick. (D) False pretenses.

D The accountant has committed theft by false pretenses. At common law, theft by false pretenses occurs when a defendant (i) obtains title; (ii) to the property of another; (iii) by an intentional (or knowing) false statement of past or existing fact; (iv) with the intent to defraud another. In the instant case, the accountant's conduct meets all of the elements of the crime. The accountant obtained title to the property by falsely misrepresenting the number of hours that he worked, with the intent that the company would pay him for the hours. Thus, he has committed the crime of false pretenses. (A) is incorrect. At common law, forgery consisted of (i) a making or altering; (ii) of a false instrument; (iii) with the intent to defraud. The falsity must be about the instrument itself, not about the contents of 2 the instrument. Here, the timecard is exactly what it purports to be—a timecard containing the hours worked by an employee. Thus, the accountant has not committed forgery by submitting a false timecard. (B) is also incorrect. At common law, embezzlement was (i) the fraudulent; (ii) conversion; (iii) of the property; (iv) of another; (v) by a person in lawful possession of that property. In the instant case, it is the last element that is lacking. Although the accountant was required to maintain the timekeeping cards for his crew and to submit budgets, he was never actually in lawful possession of any funds. Thus, he cannot properly be convicted of embezzlement. Finally, (C) is also incorrect. At common law, larceny by trick occurred when possession of property was conveyed rather than actual title. Here, the company intended to pay the accountant for the hours he purportedly worked; thus, it intended to pass title to the money the accountant received. As a result, the accountant did not commit larceny by trick.

The accused planned to hold up his neighborhood market. One day he sat at the bus stop, near the market, watching to see when would be the best time. When he saw the market was empty, he got up, walked over to the market, opened the door, and went inside. He walked up to the counter with his hand in his jacket pocket to simulate a gun. Before the clerk could turn around to see what the accused wanted, a customer entered the market, startling the accused, who turned and ran out the door. On a charge of attempted robbery, the accused should be found: (A) Not guilty, because he used no actual force on the clerk nor threatened any. (B) Not guilty, because he withdrew successfully from the robbery attempt. (C) Not guilty, because he never entered the zone of perpetration. (D) Guilty, whether or not he totally abandoned his plan when the customer entered the market.

D The accused should be found guilty of attempted robbery because, with the specific intent to commit a robbery, the accused went beyond mere preparation for the offense, and having done so, abandonment is not a defense. A criminal attempt is an act that, although done with the intention of committing a crime, falls short of completing the crime. The defendant must have the intent to perform an act and obtain a result that, if achieved, would constitute a crime. Also, the defendant must have committed an act beyond mere preparation for the offense. If a defendant has, with the required intent, gone beyond preparation, the general rule is that abandonment is not a defense. Even in those jurisdictions in which abandonment is a defense, such abandonment must be: (i) fully voluntary and not made because of the difficulty of completing the crime or because of an increased risk of apprehension; and (ii) a complete abandonment of the plan made under circumstances manifesting a renunciation of criminal purpose, not just a decision to postpone committing it or to find another victim. Here, the accused intended to take money from the clerk at the market by means of the threat of having a gun (i.e., by simulating a gun). Thus, the accused intended to commit a robbery. In walking up to the market counter while simulating a gun with his hand, the accused committed an act that was a substantial step toward commission of the intended crime, and that strongly corroborated his intent and purpose to commit the crime. All that was missing to complete the crime was for the clerk to turn around and, upon seeing the accused apparently armed, be forced to give up the money. Thus, the accused went far beyond mere preparation for the crime of robbery. Having gone beyond mere preparation, with the intent to commit robbery, the accused is guilty of attempted robbery. And, as explained above, even if the accused abandoned his plan when the customer entered the market, such abandonment will not afford him a defense. Even in those jurisdictions in which abandonment is a defense, the accused will not have a defense because his abandonment apparently occurred when the customer's sudden presence increased the risk of apprehension. Thus, the abandonment did not really manifest a renunciation of criminal purpose. (A) is incorrect because, to be guilty of attempted robbery, events need not have progressed to the point where the defendant has used or threatened to use force. Since the accused had the requisite intent for attempt and went beyond mere preparation by standing at the counter and simulating possession of a gun, he should be found guilty of attempted robbery. (B) is incorrect because, as detailed above, abandonment of an attempt does not afford a defense, and in any event, the accused's abandonment here did not really come about by way of renouncing his criminal purpose. (C) is incorrect because a conviction of attempt does not require entry into a "zone of perpetration." Rather, a defendant (with the requisite intent) need only have committed an act beyond mere preparation. The Model Penal Code and most state criminal codes require that the act constitute a substantial step towards commission of the crime and strongly corroborate the actor's criminal purpose.

A husband who believed that his wife was having an affair with his brother hired an arsonist to burn down the brother's house. They planned for the husband to take his brother to a ballgame so that the arsonist would be able to set the house on fire without detection. After the husband and brother left for the ballgame, however, the arsonist decided to abandon the plan and immediately left town without doing anything further. When the husband returned from the ballgame with the brother, he saw the house still standing and blurted out what was supposed to have happened. The husband and the arsonist were arrested and charged with conspiracy to commit arson. At the arsonist's trial, his attorney argued that he was innocent of the conspiracy because he decided not to go ahead with the plan, and nothing criminal had in fact occurred. At common law, how should a jury find the arsonist? (A) Not guilty of conspiracy, because going to a ballgame is not a criminal overt act. (B) Not guilty of conspiracy, because the husband, not the arsonist, committed the overt act. (C) Guilty, because the husband executed his part of the plan. (D) Guilty, because the arsonist agreed to set the brother's house on fire.

D The arsonist should be found guilty. A conspiracy is a combination or agreement between two or more persons to accomplish some criminal or unlawful purpose, or to accomplish a lawful act by unlawful means. The mens rea required for conspiracy is specific intent, in that both parties must intend to agree to accomplish some criminal or unlawful purpose. Thus, once the arsonist was hired by the husband and they came up with a plan to burn down the brother's house, the crime of conspiracy was completed. (C) is incorrect because it implies that carrying out the plan by at least one party is required; the conspiracy was complete even before the husband fulfilled his duties under the plan. Note that, while most states now require an overt act for conspiracy, the common law version does not. (A) is incorrect. Even if an overt act were required, it need not be in and of itself criminal. (B) is also incorrect. If an overt act were required, it need only be performed by one of the co-conspirators, not necessarily the conspirator on trial.

The defendant's neighbor owned an authentic major league baseball signed by Babe Ruth. The defendant asked if he could show it to some friends who were visiting. The neighbor agreed as long as he kept it in the display case, which the defendant promised to do. In fact, the defendant intended to use the ball in a pickup game. During the game, the ball was hit over the fence and into a yard with a guard dog, which had chewed up several other balls that had previously landed in the yard. The dog did the same to that ball. When the neighbor learned what happened to the ball, he pressed charges against the defendant. If the defendant is convicted, he will most likely be found guilty of what crime? (A) Common law larceny. (B) Embezzlement. (C) False pretenses. (D) Larceny by trick.

D The defendant is guilty of larceny by trick because he obtained possession of the baseball by means of a misrepresentation. Larceny is the taking and carrying away of tangible personal property of another by trespass, with intent to permanently (or for an unreasonable time) deprive the person of her interest in the property. The taking must be without the consent of the person in possession of the property. If such consent is induced by a misrepresentation of a past or existing fact, the consent is not valid. The resulting larceny is called larceny by trick. Here, the defendant obtained possession of the baseball with the owner's consent. However, this consent was obtained by means of the defendant's misrepresentation about friends visiting. This was a false statement of an existing fact, made with the intent that his neighbor rely on the statement, and the misrepresentation induced his neighbor's consent. At the time of this taking, the defendant intended to deal with the baseball in a manner that involved a substantial risk of damage or loss. This suffices as intent to permanently deprive. Therefore, all the elements are in place for larceny by trick. (A) is not as good a choice as (D) because the taking in this case is better characterized as larceny by trick rather than larceny, given that the defendant induced his neighbor to consent to his taking possession of the baseball. (C) is incorrect because the defendant obtained only possession of the baseball, not title. False pretenses differs from larceny by trick in what is obtained. If the defendant obtains only possession of the property, the offense is larceny by trick, whereas obtaining of title means that false pretenses has been committed. What the victim intended to convey to the defendant is determinative. The neighbor intended only to let the defendant borrow the baseball for a short time, not to convey title to him. Consequently, the only thing the defendant obtained was possession of the baseball. Because title to the baseball was not obtained, there can be no conviction of false pretenses. Regarding (B), embezzlement is the fraudulent conversion of property of another by a person in lawful possession of that property. In embezzlement, misappropriation occurs while the defendant has lawful possession of the property, while in larceny, it occurs generally at the time the defendant obtains wrongful possession of the property. Here, as detailed above, the defendant's taking of possession of the 13 baseball was trespassory due to the manner in which he obtained consent to such possession. The crime of larceny was complete on the defendant's taking possession with the requisite intent to permanently deprive. Thus, at the time the baseball was destroyed, the defendant had already misappropriated it and was not in lawful possession of it. As a result, there can be no conviction for embezzlement.

The defendant's neighbor owned an authentic major league baseball signed by Babe Ruth. The defendant asked if he could show it to some friends who were visiting. The neighbor agreed as long as he kept it in the display case, which the defendant promised to do. In fact, the defendant intended to use the ball in a pickup game. During the game, the ball was hit over the fence and into a yard with a guard dog, which had chewed up several other balls that had previously landed in the yard. The dog did the same to that ball. When the neighbor learned what happened to the ball, he pressed charges against the defendant. If the defendant is convicted, he will most likely be found guilty of what crime? A. Common law larceny. B. Embezzlement. C. False pretenses. D. Larceny by trick.

D The defendant is guilty of larceny by trick because he obtained possession of the baseball by means of a misrepresentation. Larceny is the taking and carrying away of tangible personal property of another by trespass, with intent to permanently (or for an unreasonable time) deprive the person of her interest in the property. The taking must be without the consent of the person in possession of the property. If such consent is induced by a misrepresentation of a past or existing fact, the consent is not valid. The resulting larceny is called larceny by trick. Here, the defendant obtained possession of the baseball with the owner's consent. However, this consent was obtained by means of the defendant's misrepresentation about friends visiting. This was a false statement of an existing fact, made with the intent that his neighbor rely on the statement, and the misrepresentation induced his neighbor's consent. At the time of this taking, the defendant intended to deal with the baseball in a manner that involved a substantial risk of damage or loss. This suffices as intent to permanently deprive. Therefore, all the elements are in place for larceny by trick. (A) is not as good a choice as (D) because the taking in this case is better characterized as larceny by trick rather than larceny, given that the defendant induced his neighbor to consent to his taking possession of the baseball. (C) is incorrect because the defendant obtained only possession of the baseball, not title. False pretenses differs from larceny by trick in what is obtained. If the defendant obtains only possession of the property, the offense is larceny by trick, whereas obtaining of title means that false pretenses has been committed. What the victim intended to convey to the defendant is determinative. The neighbor intended only to let the defendant borrow the baseball for a short time, not to convey title to him. Consequently, the only thing the defendant obtained was possession of the baseball. Because title to the baseball was not obtained, there can be no conviction of false pretenses. Regarding (B), embezzlement is the fraudulent conversion of property of another by a person in lawful possession of that property. In embezzlement, misappropriation occurs while the defendant has lawful possession of the property, while in larceny, it occurs generally at the time the defendant obtains wrongful possession of the property. Here, as detailed above, the defendant's taking of possession of the baseball was trespassory due to the manner in which he obtained consent to such possession. The crime of larceny was complete on the defendant's taking possession with the requisite intent to permanently deprive. Thus, at the time the baseball was destroyed, the defendant had already misappropriated it and was not in lawful possession of it. As a result, there can be no conviction for embezzlement.

The defendant wanted to borrow his neighbor's car to go shopping for lawnmowers. Knowing that the neighbor was out of town for the weekend, the defendant opened the neighbor's garage door and took a car key that the neighbor hid in an old coffee can for emergencies. Once inside, the defendant, mistakenly believing that larceny applied only to the taking of items valued at over $100, decided to take the neighbor's lawnmower (worth $75) in order to trade it in on a new mower. However, on the way to the store with the mower, he was involved in an automobile accident, totaling the car. In a common law jurisdiction, of what property is the defendant guilty of larceny? (A) Both the car and the mower. (B) Neither the car nor the mower. (C) The car, but not the mower. (D) The mower, but not the car.

D The defendant is guilty of larceny of the mower, but not the car. Larceny is the taking and carrying away of the personal property of another by trespass with the intent to permanently (or for an unreasonable time) deprive the other of his interest in the property. The intent to permanently deprive may be found when the defendant intends to use the property in such a manner as to create a substantial risk of loss. As to the mower, the defendant took and carried away the mower with the intent to permanently deprive his neighbor of it. The defendant's mistake as to the coverage of the criminal law does not negate his intent to commit the crime, and thus provides no mistake of law defense. Thus, (B) and (C) are incorrect. However, the defendant is not guilty of larceny of his neighbor's car because he did not have the intent to permanently deprive the neighbor of the car; rather, his intent was to borrow the car. Furthermore, merely driving a car does not constitute a use that creates a substantial risk of loss sufficient to find the intent to permanently deprive the neighbor of the car. As a result, (A) and (B) are incorrect.

The defendant pointed a loaded gun and shot at the victim. The bullet, however, missed the intended victim, but struck a bystander in the stomach. The bystander fully recovered. In a subsequent prosecution for attempted murder of both the victim and the bystander, the defendant testified that he had wanted only to scare the victim. Assuming that the jury believes this testimony, the defendant may be convicted of attempted murder as to whom? A. The victim. B. The bystander. C. Both the victim and the bystander. D. Neither the victim nor the bystander.

D The defendant may not be convicted of attempted murder of the victim or bystander because he lacked the necessary intent. A criminal attempt consists of: (i) conduct that brings the defendant in close proximity to the completed offense; and (ii) the intent to commit the completed crime. In other words, the defendant must have the intent to perform an act and obtain a result that would constitute the crime charged if achieved. Regardless of the intent required for the completed offense, an attempt always required specific intent. Thus, attempted murder required the specific intent to kill another person, even though the mens rea for murder itself does not require specific intent—had the bystander died, the defendant could have been convicted of murder, given that malice could have been established by the defendant being aware of an unjustifiably high risk to human life (i.e., an "abandoned and malignant" heart) by pointing a loaded gun and shooting at an individual. However, the defendant did not have the intent to kill either victim, so he lacked the intent necessary for attempt. (D) is therefore correct, and (A), (B), and (C) are incorrect. In answering questions such as this one, remember to be objective and answer the question asked. Although the defendant here is surely guilty of some crimes—e.g., assault and battery—he is not guilty of the charged crime.

A wife suffered from a particularly virulent form of cancer, and had lapsed into a nearly comatose state. Because the doctors had indicated that any treatment they could prescribe would be of little value, her husband decided to administer various poisons to his wife, thinking that they might stimulate her natural body defenses, or kill the cancer cells, resulting in her recovery. He tried doses of many different types of poison. Despite his ministrations, his wife died three days later. An autopsy performed by the county coroner established the cause of death as cancer. If the husband is prosecuted for the murder of his wife, which of these is the best reason why he would be acquitted? A. He was trying to save her life. B. He did not have the necessary malice for his actions to constitute murder. C. Medical science had given her up for dead. D. He did not cause her death.

D The husband cannot be convicted because he did not cause his wife's death. Murder is defined as the unlawful killing of another human being with malice aforethought. To be guilty of murder, the defendant's action must be both the cause in fact and the proximate cause of the victim's death. The defendant's act will be a cause in fact of death if, but for the defendant's action, the victim would not have died as and when she did. Here, the victim would have died when she did even if the husband had not administered the poison, because she died not from the poison, but only from her cancer. Thus, the husband's actions were not the cause in fact of death, and (D) is correct. (A) and (B) are incorrect because if the other elements of murder are established, administering poison might be sufficient to establish malice aforethought. Malice aforethought for murder can be established by conduct done with the awareness of an unjustifiably high risk to human life, and the husband knew that the poisons were dangerous and could kill. (C) is incorrect because the law forbids shortening a life even for one second, so it is not a defense that medical science had given the victim up for dead. If the defendant's action in any way shortened the victim's life, he can be held liable for murder

During the nighttime, a woman broke into the house of the victim with the intention of stealing his diamond ring. When she could not find the diamond ring, she became angry, lit a match to a newspaper and threw it on the victim's bed, setting the mattress on fire. The flames destroyed the bed and a portion of the floor under the bed. In a common law jurisdiction, of which crime or crimes is the woman guilty? A. Burglary only. B. Arson only. C. Burglary and attempted arson. D. Burglary and arson.

D The woman is guilty of arson and burglary. She is guilty of arson because she deliberately set a fire that, in addition to burning the mattress, also burned part of the dwelling house of another, namely the floor. She is also guilty of burglary because she broke and entered into the dwelling house of another during the nighttime to commit a felony. The fact that she was not successful in committing the crime she intended or that she in fact committed another felony is irrelevant to her guilt for burglary; it is the intent to commit a felony at the time of the breaking and entering which is critical. (A) is incorrect. This choice correctly states that the woman is guilty of burglary, but she is also guilty of arson; she deliberately set a fire that, in addition to burning the mattress, also burned part of the dwelling house of another, namely the floor. Therefore, (B) and (C) are incorrect.

A woman whose car was in the shop wanted to meet her friend at a restaurant a mile away from the woman's home. She noticed that her neighbor had forgotten to lock his bicycle upon returning home. The woman took her neighbor's bicycle and began riding it to the restaurant, intending to bring it back in the trunk of her friend's car after her dinner. She had ridden the bicycle a distance of two blocks when she was hit by a car that had gone through a red light. The bicycle was destroyed in the collision. If the woman is charged with larceny of the bicycle, how will she most likely be found? A. Guilty, because she intended to take the bicycle without permission. B. Guilty, because she was responsible for the permanent loss of the bicycle. C. Not guilty, because she did not ride the bicycle far enough away from where she took it. D. Not guilty, because what she planned to do with the bicycle did not create a substantial risk of permanent loss.

D The woman will most likely be found not guilty, because what she planned to do with the bicycle did not create a substantial risk of permanent loss. To be guilty of larceny, a defendant must intend to steal, which can be established either by intending to permanently deprive or intending to create a substantial risk of loss of the item involved. The woman intended to take the bicycle, but she also intended to safely return it. Thus, she did not have the specific mental state necessary to be guilty of larceny. (A) is therefore incorrect. (B) is incorrect because the key factor is the defendant's mental state, not what actually happened to the bicycle. (C) is incorrect because it does not address the intent requirement. Furthermore, it incorrectly implies that some specific distance is required to satisfy the asportation requirement for larceny; in fact, any slight movement of the property will be sufficient.


संबंधित स्टडी सेट्स

Tests/quizzes Service and Maintenance

View Set

anatomy muscular system test review

View Set

CHAPTER 4 - GOVERNMENT LOANS - FHA/VA/USDA Study Guide

View Set

QUIZ: Chapter 9: Constructing Gender and Sexuality

View Set